Anda di halaman 1dari 55

1 A 27 years old woman, 32 weeks’ gestation complains of shortness of breath during her

pregnancy, especially with physical exertion. She has no prior medical history. Her
respiratory rate is 16. Lung auscultation are clear and her oxygen saturation to be 98% on
room air.

You reassure her that this sensation is normal and explain which of the following? E
a. Because of enlarging uterus pushing up on the diaphragm, her vital capacity is decreased
by 20%.
b. Airway conductance is decreased during pregnancy.
c. Pulmonary resistance increases during pregnancy.
d. Small amniotic fluid emboli are shed throughout pregnancy.
e. Maximal breathing capacity is not altered by pregnancy

2 A 30-year-old woman comes to your practice desiring pregnancy. She has a history of
regular, 28-day cycles. She has had two pregnancies in the past, one ending in miscarriage
at 9 weeks and one vaginal delivery at 37 weeks. After she delivered the baby, she using
intra uterine device for contraception, and now she is planning to become pregnant.

Nutritional supplements she should begin before she gets pregnant include which of the C
following:
a. Vitamin B1 to reduce beriberi
b. Vitamin B12 to increase RBC production prior to pregnancy
c. Folate to reduce neural tube defects
d. Vitamin C to reduce scurvy
e. No supplementation is necessary until pregnancy is confirmed

3 A 28-year-old G1P0A0 woman at 38 weeks and 5 days presents to labor and delivery unit,
with regular contractions occurring 3-4x/10 minutes. Her contractions last 30 to 90 seconds.
On vaginal examination you found 10 cm dilatation and now in the second stage of labor.
She is pushing effectively, but during contractions you notice decelerations on fetal heart
tracings.

Which of the following would be most dangerous? E


a. Absence of decelerations
b. Isolated early decelerations
c. Repetitive variable decelerations that resolve quickly after each contraction
d. Repetitive early decelerations and variable decelerations
e. Repetitive late decelerations and loss of variability between contractions

4 A 30-year-old G3P1A1 at 33 weeks’ gestation reports some recent regular contractions.

Which of the following correlates with the greatest risk for preterm labor? D
a. History of colposcopy
b. Prior 8-week spontaneous abortion
c. History of Chlamydia trachomatis
d. Prior 32-week delivery
e. Patient is a smoker (half pack per day)

5 A 41-year-old primigravida presents to your office during the ninth week of pregnancy. B
Because of her age, so they are extremely anxious and would like prenatal diagnosis as
early as possible. They have many questions about chorionic villus sampling (CVS)

You explain to the couple that complications of CVS include which of the following?
a. Amniotic band syndrome
b. Vaginal bleeding
c. VATER syndrome
d. IUGR
e. Rupture of the umbilical cord

6 A 26-year-old G2P1A0 comes to you at 16 weeks’ gestation. She is currently on valproic acid
to control her epilepsy with grand mal seizures that developed after motor vehicle trauma.

What risk associated with valproic acid use for her fetus? C
a. Hypospadias
b. Aplasia actuis
c. Neural tube defect
d. Club feet
e. Diaphragmatic hernia

7 A 28-year-old G1P0A0 woman presents at 11 weeks’ gestation for routine prenatal care. She
has no complaint and there is no important past medical history. She is interested in first-
trimester screening.

Which of the following is true about first-trimester screening? C


a. First-trimester serum screening has an 80% sensitivity for Down syndrome
b. Combined first-trimester screening has a 90% sensitivity for Down syndrome
c. NT screening alone has a 70% sensitivity for Down syndrome
d. First-trimester screening has a sensitivity for Down syndrome that is greater than the
sequential screen
e. Increased NT in the setting of normal karyotype is associated with limb anomalies

8 A 35-year-old G3P2 woman at 39 weeks and 4 days was admitted to labor and delivery unit
for active management of labor. After her membranes had ruptured, she is very
uncomfortable and finds her contractions very painful. You perform cervix examination is at 6
cm.

Which of the following presentations and positions would you expected be most favorable to A
achieve a vaginal delivery?
a. Vertex with occiput anterior
b. Vertex with occiput posterior
c. Vertex with occiput transverse
d. Breech with sacrum anterior
e. Breech with sacrum posterior

9 A 38-year-old woman G6P5A0 is in the active phase of labor with the vertex at -1 station.
She complains of abdominal pain with the contractions. At the height of one contraction, the
pain becomes very intense. Following this intense pain, uterine contractions stop.The
maternal BP drops from 120/80 to 90/60 mmHg.
What is the best first step of action? D
a. Place the patient on her side and reassure her
b. Perform cardiotocography
c. Begin oxytocin to increasing contraction
d. Immediately perform a pelvic examination
e. Perform C-section immediately

10 A 22-year-old G2P1 woman at 38 weeks by LMP presents to labor and delivery (L & D)
triage, complaining of painful uterine contractions 3x/10 minutes. Her obstetrical history is
significant for one prior low-transverse cesarean section for breech at 39 weeks’ gestation.
On sterile vaginal examination, the patient is initially found to be 5 cm dilated, 75% effaced,
and -2 station. The patient strongly desires to have a vaginal delivery if at all possible.

What is next best step? C


a. Discharge home and follow-up in 1 week at clinic
b. Administer tocolysis to stop painful uterine contraction
c. Admission to L & D triage, continuous fetal monitoring, counsel patient regarding trial of
labor after cesarean (TOLAC), proceed with expectant management of labor if she desires to
proceed with TOLAC
d. Admission to L & D triage, continuous fetal monitoring counsel patient regarding TOLAC,
and start oxytocin augmentation if she desires to proceed with TOLAC
e. Admission to L & D triage and repeat cesarean delivery without a trial of labor

11 A 38-year-old woman presents with heavy menses. She is found an enlarged uterus on
pelvic examination. Ultrasound reveals a well-circumscribed intramural mass consistent with
the leiomyoma.

You explain the incidence of sarcomatous degeneration in a uterine leiomyoma is ... A


a. <1%
b. 3%
c. 10%
d. 15%
e. 30%

12 A preeclampsia patient referred from midwife to your emergency room. From Her prenatal
care it appears that she is at 37 weeks’ gestation. She has no complain and her BP is
170/110 with proteinuria ++. You plan to give her magnesium sulfate. Magnesium sulfate has
potential for serious complications such as respiratory depression, hypoxia, cardiac arrest,
and even death.

Which of the following clinical tests is most useful to monitor patients who are receiving E
magnesium sulfate infusions?
a. Serial BP measurements
b. Serum magnesium levels every 2 hours
c. Serum magnesium levels every 4 hours
d. Serial pulmonary examination
e. Serial deep tendon reflex examination

13 A 33-year-old G2P1 woman at 10 weeks’ gestation presents to clinic for routine prenatal visit.
Her pregnancy has been uncomplicated. Her prior pregnancy 2 years ago was complicated
by the diagnosis of gestational hypertension that led to an induction of labor. She has no
other medical history. On examination her BP is 138/88 mm Hg, her body mass index (BMI)
is 37 kg/m, and a urine dip shows trace protein. The patient’s 24-hour urine shows 100 mg of
protein. You counsel the patient on the pregnancy complications associated with chronic
hypertension as well as management.

What is the next best step in her management? E


a. Recommend termination of pregnancy
b. Start antihypertensive therapy
c. Bed rest
d. Hospitalization for further maternal and fetal testing
e. Expectant management with close observation for early signs of preeclampsia and fetal
growth restriction

14 A 30-year-old G2P1 woman at 40 weeks 5 days presents to labor and delivery triage with a
5-hour history of painful contractions. In her first pregnancy, A 4,200 g baby was delivered by
cesarean section after arrest at 7 cm for 4 hours. Today your patient requests a trial of labor
after cesarean section. Monitoring reveals contractions every 3 minutes and cervical
examination on arrival is 4 cm dilated, 90% effaced, and -1 station. Her pregnancy has been
complicated by gestational diabetes and 20.4 kg weight gain in pregnancy (BMI 24). A recent
ultrasound preformed at 38 weeks estimated the fetal weight at 3,900 g. You admit the
patient to labor and delivery for expectant management.

Which of the following factors places the patient at greatest risk for shoulder dystocia? D
a. Prior failure to progress
b. Prior fetus weighing 4,200 g
c. 20.4 kg weight gain in pregnancy
d. Suspected fetal macrosomia
e. Gestational diabetes

15 A 33-year-old woman complains of pelvic pain and secondary dysmenorrhea that is


increasing in severity. The pain is triggered by deep penetration with coitus.

Which of the following is the most common cause of deep-penetration dyspareunia? A


a. Endometriosis
b. Depression
c. Vaginismus
d. Vestibulitis
e. Atrophic change

16 A 21-year-old G1P0 woman presents at 25 weeks’ gestation complaining of headache for the
past 36 hours. She has had regular prenatal visits since first trimester. A 20-week ultrasound
redated her pregnancy by 2 weeks as it was 15 days earlier than her LMP dating. She has a
BP of 150/105 mm Hg. She does not have chronic hypertension. The patient denies having
right upper quadran (RUQ) pain but because of your high suspicion of severe preeclampsia
you order a CBC, liver enzymes, renal function test, and a 24-hour urine protein collection.
Her laboratory test results reveal a normal platelet count and liver enzymes. Protein urine
level is 600 mg in 24 hours. Her headache has resolved after a dose of acetaminophen.

What is the next best step in her management? E


a. Begin induction of labor
b. Give her a prescription for labetalol and have her follow-up in clinic in 2 weeks 


c. Immediate delivery
d. Give her nifedipine and have her follow-up in clinic in 1 weeks

e. Hospitalization for further evaluation and treatment

17 A 34-year-old G2P1 woman with obesity came to your clinic. She presents at approximately
10 weeks’ estimated gestational age. Her BMI was 32 kg/m2 before get pregnant. In her
previous pregnancy, she required insulin therapy for GDM, and she has family history for
type 2 diabetes mellitus (T2DM). She delivered at 38 weeks and her baby boy weighed 4,500
g.

In addition to the routine prenatal laboratory tests, what other testing do you also obtain at this D
time?
a. An ultrasound to estimate fetal weight 

b. Random Blood Glucose
c. The best time to perform glucose challenge test is 
at 24-28 weeks’ gestation, so no testing
needed at this time
d. A 75-g load glucose challenge test with fasting and a 2-hour blood glucose
e. She has GDM so no other testing needed.

18 A 45-year-old P0A0 patient with prolonged menstruation and dysmenorrhea. From pelvic
examination found enlarged uterus, both adnexa within normal limit.

Which study listed below would best differentiate between adenomyosis and uterine B
fibroids?
a. Pelvic ultrasound 

b. Pelvic MRI 

c. Pelvic CT 

d. Sonohysterogram 

e. Hysterosalpingogram

19 A 28-year-old P0A0 woman presents a low-grade fever and generalized malaise that is
improved. But now she complaints of painful genital ulcer for two days. She denies any
history of genital ulcers. She has had three sexual partners in the last year and uses
condoms inconsistently. Several painful vesicles are noted on the left labia minora.

What kind of microorganism of the following is the most likely cause of this disease? C
a. Trichomonas vaginalis
b. Treponema pallidum
c. Herpes simplex virus
d. Haemophilus ducreyi
e. Chlamydia trachomatis

20 A 22 year-old woman complains of a profuse nonirritating discharge. From speculum


examination presented of thin, white, homogeneous discharge coating the vaginal walls. An
amine odor noted with “whiff” test and pH greater than 4.5.

What most common type of vaginitis with a high pH? A


a. Bacterial vaginosis
b. Candidal vaginal infections
c. Mucopurulent cervicitis
d. Trichomonas
e. Atrophic vaginitis

21 A 29-year-old P1A0 woman who suffered from a low-grade fever of 38.0°C at post-partum
day 1 come to your outpatient-clinic. She is now attempting to breastfeed her baby but lack of
volume. She knows the benefits of breastfeeding and continues to try. Her pregnancy and
delivery were uncomplicated. She complains about the pain associated with the engorgement
of her breasts bilaterally and very sharp, recurrent pelvic pains. Her vital signs are otherwise
normal.

What is the most likely explanation for these findings? D


a. Chorioamnionitis
b. Endometritis 

c. Breast abscess 

d. Lactation fever 

e. Breast ulcer 


22 A 25-year-old G3P1A1 woman referred from primary health care due to HIV. Her pregnancy
is at 10 weeks’ GA. She had never been on HAART medication before this pregnancy. You
order a viral load and it returns at 12,000. Her CD4 count is normal (600). She is otherwise
healthy and has no other significant medial history. You start her on a three-drug HAART
regimen of zidovudine, lamivudine, and lopinavir/ritonavir at 14 weeks. Her viral load was
decreased in the second and third trimester, 5.000 and 2,250. At 39 weeks she is admitted to
labor and delivery in active labor.

What would you recommend to her in order to further decrease neonatal transmission? A
a. Cesarean delivery 

b. Early artificial ROM for labor augmentation 

c. Stopping all antiviral therapy to decrease risk of neonatal 
resistance 

d. Assisted vaginal delivery with vacuum or forceps
e. Placement of a fetal scalp electrode to monitor for signs of 
fetal distress 


23 A 60-year-old woman is referred by her GP with the complaint of ’something coming down in
the vagina’ as well as feeling a lump. She has no complaint about mixturition. Physical
examination found decent of uterine prolapse until the level of the hymen.

Using the Baden-WalkerHalfway System, this prolapse is considered which of the following? B
a. Grade 1
b. Grade 2
c. Grade 3
d. Grade 4
e. Grade 0 


24 A 58-year-old P4A0 woman is referred to the urogynecology clinic for urinary leakage by her
GP. She complains urine loss with coughing, sneezing, straining and she uses 2-3 pads per
day for urinary leakage for the past 2 years. Leakage of urine with urgency was denied. Her
examination is remarkable for obesity. After trying behavioral and lifestyle modifications, the
patient continues to be symptomatic. Urinalysis and urine cultures result were negative.
Urodynamics studies is performed. Cystometry reveals leakage of urine with increase in
intrabdominal pressure during valsava effort and coughing. No involuntary detrusor
contraction are seen. Bladder compliance is normal. The pressure flow pattern is continuous
and normal with a void initiated by urethral relaxation and a normal detrusor contraction.

What type of urinary incontinence does this patient most likely have? D
a. Urgency incontinence 

b. Functional incontinence 

c. Mixed incontinence 

d. Stress incontinence 

e. Overflow incontinence 


25 A 22-year-old P0A0 patient comes to your clinic for a complaint of a mass located on her
vulva near the introitus. It seems to be aggravated by intercourse, but usually goes away on
its own. No pain at that mass. She is had only one boyfriend and did intercourse without
condoms. She has never had an STI. You examine her and a 4 cm nontender mass at
approximately 4-o’clock positions on the posterior-lateral aspect of the vaginal orifice.

What is the most likely diagnosis? C


a. Skene’s gland cyst 

b. Gartner’s duct cyst 

c. Bartholin’s duct cyst 

d. Cystocele 

e. Epidermal inclusion cyst 


26 A 28 year-old mother delivered her infant with abnormalities. During her pregnancy, she had
an episode of fever and painful swallowing in the first trimester. When she did antenatal care,
GP said that her uterine fundal height was less than 2 weeks than expected so she must
performed ultrasound but she didn’t performed. Now her infant is small for gestational age
and demonstrates microcephaly, chorioretinitis, deafness, and delayed development later in
life.

Which of the following is the most likely cause? B


a. Late-onset group B streptococcal infection
b. CMV infection during pregnancy
c. Parvovirus infection
d. Type 2 herpes hominis virus acquired at the time of delivery
e. Vitamin K deficiency in the newborn

27 A 24 year old women come to your clinic with her spouse. They have been married for 2
years and haven’t been pregnant yet. The doctor perform ultrasound examination and told
them that it might be because of genital abnormality.

Which of the following is the result of lack of fusion of the Müllerian duct system? C
a. Unilateral renal agenesis
b. Transverse vaginal septum
c. Uterine didelphys
d. Gardner cyst
e. Imperforate hymen
28 A 55-year-old woman P5A2 comes in complaining that she has noticed a bulge protruding
from her vagina. She has a family history of genital prolapse. On examination, you notice a
uterine prolapse, cystocele, and rectocele.

Which of the following ligaments provide the most support to the uterus in terms of E
preventing prolapse?
a. Utero-ovarian ligaments
b. Arcuate ligament
c. Round ligaments

d. Broad ligaments

e. Cardinal ligaments


29 You are an OBGYN in outpatient clinic today. Your patient a 27-year-old woman who has
presented with prolonged menstrual bleeding since 1 year. During the evaluation this patient
has normal blood pressure and heart beat. A urine pregnancy test is negative. Based on
ultrasound examination, you suspected uterine fibroid.

Which of the following fibroid locations is most commonly associated with abnormal vaginal A
bleeding ?
a. Submucosal

b. Subserosal

c. Intramural 

d. Serous - Pedunculated
e. Parasitic


30 A 55-year-old G3P2 presents with postmenopausal bleeding. Transvaginal ultrasound


reveals a 13-mm endometrial stripe and an 9-cm complex left adnexal mass. An endometrial
biopsy reveals endometrial hyperplasia.

What characteristics of the adnexal mass make it concerning for malignancy? 
 E


a. Fixed, non mobile location 

b. Internal septations
c. Ascites with peritoneal masses
d. Papillary and nodular solid components 

e. All of the above

31 A 28-year-old P2A0 healthy woman come to your outpatient clinic for consultation about risks
of her getting breast cancer. Her mother developed breast cancer at age 42, and died at 48.
Her sister had ovarian cancer at age 35. She wants to know if there are any tests she should
get to help her evaluate her risks.

Which of the following tests would most likely you will perform? B
a. Decreased T- and B-cell levels cause a decline in immunity.
b. BRCA1 and BRCA2 mutations may predict increased risk of breast cancer.
c. Human epidermal growth factor receptor 2 (HER-2)/ neu gene levels are associated with
later development of breast cancer.
d. Serum androgen levels are associated with a decreased risk of breast cancer.
e. CA-125 is a good prognostic marker for breast cancer.
32 A 24-year-old G1P0 at 13 weeks’ gestation who has sickle-cell anemia requests genetic
counseling to learn about the risk of transmission to her fetus.

If her partner is a heterozygous carrier for this condition, what is the risk that their offspring B
will be affected?
a. 25%
b. 50%
c. 75%
d. 80%
e. 100%

33 Mr. and Mrs. S counseling about infertility. They had been married for 5 years. They are
planning to have an IVF.

In your discussion about conception, tubal disease, and implantation, you explain to them that B
implantation in the uterus occurs at which stage of development?
a. Eight-cell embryo
b. Blastocyst
c. Zygote
d. Morula formation
e. Embryonic disk

34 Mrs. Y, 37 yo, G2P1 37 wga desire VBAC (vaginal birth after cesarean section), but she
worries about its complication. Her baby is in a vertex presentation and her previous low
transverse procedure was for postterm and failed of induction.

In providing informed consent, in which of the following ways do you explain the risk of uterine A
rupture?
a. Less than 1%
b. Between 2% and 5%
c. Dependent on the length of her labor
d. 15–20%
e. Dependent on the location and proximity of the scar site to the placental implantation

35 A woman, 26 yo, G1 6 wga based on her last mens period, counseling about an alcohol
binge she had.

You explain that, with the exception of the brain, organogenesis is completed within how C
many weeks after her last period? 

a. 4 weeks after ovulation
b. 8 weeks after ovulation
c. 10 weeks after ovulation
d. 18 weeks after ovulation
e. 26 weeks after ovulation

36 A 41-year-old P2 woman comes to see you with a complaint of 14 months of “irregular


bleeding.” She previously had regular menses but over the past 2 years they have become
totally unpredictable. The doctor said that it might be caused by hormonal imbalance.

Which of the following hormones can have a negative as well as a positive feedback at the C
level of the pituitary?
a. Cortisol
b. Oxytocin
c. Estradiol
d. Progesterone
e. Insulin

37 A 23 year old G1 woman at 8 weeks’ GA comes to your clinic for an initial prenatal visit. She
had a history of cardiac disease since 5 years ago, but now she didn’t have any complaint
and not taking any medication since 1 year ago. She asks if she could continue her
pregnancy, since many people said that pregnancy could put her in danger. You explain that
it may be due to the increase of blood volume and cardiac changes in pregnancy.

The increase in blood volume in normal pregnancy is made up of which of the following? C
a. Erythrocytes
b. More erythrocytes than plasma
c. More plasma than erythrocytes
d. Neither plasma nor erythrocytes
e. Plasma only

38 A 27-year-old G2P1 woman at 8 weeks’ GA comes to your clinic for an initial prenatal visit. A
medical history reveals a diagnosis of lupus 2 years ago. She is in normal condition right
now.

What is the most significant neonatal complication of maternal lupus? C


a. Congenital abnormalities 

b. Neonatal thrombosis 

c. Neonatal heart block 

d. Acute respiratory distress syndrome (ARDS) 

e. Feeding difficulties 


39 A 28-year-old woman and her partner present to their physician as they have not been able
to achieve pregnancy during the past 1 year. Her partner, age 35, and she are both healthy
and take no prescription medications. They are sexually active two to three times per week
and do not use any contraception.

Prior to initiating a potentially expensive infertility evaluation, they ask what percentage of C
reproductive-age couples are unable to conceive after 1 year of coitus without
contraception? 

a. 5
b. 10
c. 15
d. 35
e. 60 

40 Mrs. S, 34 yo came to your office six days after total abdominal hysterectomy through a
midline vertical incision, she presents with an erythematous wound with scant purulent
drainage. The wound spontaneously opens and copious foul serosanguinous fluid spills. Her
temperature is 38.2◦C. She is taken to the operating room for debridement of her incision and
examination of the fascia. Integrity of the fascial incision is documented.

Antibiotics selected for this patient should cover which of the following pathogens? E
a. Solely gram-positive bacteria 

b. Solely gram-negative bacteria 

c. Anaerobic and gram-positive bacteria 

d. Solely Anaerobic

e. Anaerobic, gram-positive and gram-negative bacteria

41 Mrs. X, 52 yo complaining about involuntary loss of urine preceded by a strong sensation to


void.

Which of the following forms of incontinence reflects her symptom ? C


a. Stress incontinence 

b. Over flow incontinence
c. Urge incontinence
d. Functional incontinence 

e. Mixed Incontinence

42 A 38-year-old P3 woman comes to you for a second opinion, she complaining about heavy
menses and worsening dysmenorrhea since 12 months. She underwent a pelvic ultrasound
that suggested adenomyosis and her gynecologist recommended a hysterectomy, but she
states that she does not trust the ultrasound results and wants to know if there is anything
else that can be done to confirm the diagnosis.

What would be the most appropriate next step? C


a. Review the ultrasound results and reassure her that her gynecologist is correct
b. Repeat the pelvic ultrasound 

c. Examine her and recommend obtaining a pelvic MRI
d. Tell her that hysterectomy is the only thing that will help to 
clarify her diagnosis
e. Suggest a 3-month trial of an oral contraceptive pill 


43 An 18-year-old nulliparous woman comes into your office, complaining of a 24-hour history of
urinary frequency, urgency, and suprapubic pain. She had intercourse recently and using a
diaphragm for birth control.

Which of the following statements reflects current knowledge about uncomplicated urinary B
tract infections?
a. Cystitis occurs two times more commonly in men than in women.

b. In women who have frequent UTIs related to coitus, infection can be prevented with low-
dose postcoital antibiotics.
c. In a freshly voided, clean-catch urine specimen, leukocyte esterase and nitrites, in
combination have a poor test performance in diagnosing UTI.

d. Use of a diaphragm helps prevent the development of UTIs.

e. A teenager is more likely to have asymptomatic bacteriuria than is a postmenopausal
woman.

44 A 39-year-old woman is reviewed in the gynecology clinic with symptoms of pelvic pain for
the last 2 years. Her ultrasound scan 2 months ago revealed a 
 left-sided ovarian cyst (6
cm) with diffuse low-level internal echoes with one thin internal septae. After performed
laparoscopy, lesions are biopsied that are thought to represent endometriosis.

The diagnosis of endometriosis is confirmed histologically by identifying extragenital implants A


containing which of the following?
a. Endometrial glands and stroma
b. Hypertrophic smooth muscle
c. Hemorrhage and iron pigment deposits
d.Fibrosis
e. Stromal decidualization

45 A 31-year-old patient infertile for 4 years and preferred for start in vitro fertilization (IVF)
because of obstructed fallopian tubes. On hysterosalpin-gogram (HSG), it is noted that she
has large dilated hydrosalpinges present bilaterally.

What should be your next step? E


a. The patient should begin her IVF treatment cycle.
b. The patient should repeat the HSG to confirm the result.
c. The patient should not be offered the opportunity to have IVF.
d. Her hydrosalpinges should be drained via transvaginal aspiration prior to starting IVF.
e. Bilateral salpingectomies should be done prior to starting IVF.

46 A 27-year-old nulligravida presents with 6 month of amenorrhea and diagnosed with


hyperprolactinemia.

Laboratory test measuring which of the following should also be obtained? D


a. Total Testosterone
b. 24-hour urinary free cortisol
c. Insulin-like growth factor II
d. Thyroid Stimulating Hormon
e. Oxytocin

47 A woman, 26 yo, attends clinic for preconception counseling. She admitting previously being
treated for breast carcinoma and planning to get pregnant.

She asking you how long after completion of the treatment is it recommended to wait before C
conceiving?
a. 6 month
b. 1 year

c. 2 years
d. 3 years
e. 5 years

48 A 40-year-old pregnant patient calls to discuss her Pap smear results. Her Pap smear shows
atypical squamous cells of undetermined significance, and human papillomavirus testing is
negative.

Which of the following is your recommendation for subsequent evaluation? 
 D


a. Colposcopy 6 weeks postpartum

b. Colposcopy in the third trimester

c. Colposcopy as soon as possible
d. Repeat cytology 6 weeks postpartum
e. Repeat cytology in the third trimester 


49 A term infant is delivered via cesarean delivery as a double-footling breech. It is noted to


have an Apgar score of 3 at 1 minute and later to be irritable and restless. The infant’s
muscles are rigid, and the anterior fontanel bulges, then progressive bradycardia.

What is the most likely cause of these findings?
 A


a. Intracranial hemorrhage
b. Brain stem injury

c. Congenital abnormality
d. Neonatal sepsis

e. Infection


50 A 51-year-old woman presents to your office due to amenorrhea for the past year but she
denied had any hot flashes, she is wondering if she is menopausal.

She asking you how to make sure, what blood test would confirm the diagnosis of C
menopause?
a. Progesteron 

b. Testosterone 

c. FSH 

d. Human chorionic gonadotropin (hCG) 

e. Prolactin
51 An 18-year old girl presents to your office with chief complain never had menstruation before.

Which of the following conditions is considered the most frequent cause of primary D
amenorrhea?
a. Asherman syndrome 

b. Androgen insensitivity syndrome 

c. Mayer-Rokitansky-Kuster-Hauser syndrome
d. Turner syndrome 

e. Congenital Adrenal Hyperplasia


52 A 37-year-old woman presents to your office with a positive urine pregnancy test and a 3-day
history of vaginal bleeding. On examination, theuterine fundus is 1 finger above the
umbilicus. By her last period, she should be around 10 weeks of gestation. On pelvic
examination, there is a moderate amount of blood and vesicle-like tissue in the vaginal vault,
and the cervix is closed. The lab then calls you to say that her serum β-hCG result is greater
than 1.500.000 mIU/mL.

Which of the following is the best next step in this patient’s evaluation? C
a. Determination of Rh status
b. Surgical intervention (suction curettage)
c. Complete pelvic ultrasound 

d. Methotrexate administration 

e. Schedule a follow-up visit in 2 to 4 weeks to recheck a β-hCG level 


53 A 39-year-old G3P2 woman presents to your emergency department with a 6-months


irregular bleeding and a new onset of coughing up blood. Her history reveals two term
vaginal deliveries, her last being approximately 6 months ago. That delivery was
uncomplicated. On physical examination, vital signs are stable, her uterus is approximately
10 to 12 weeks size, and there is a moderate amount of blood in the vaginal vault. The
quantitative serum β-hCG is 500.000 mIU/ml. Already performed curretage, with PA result
choriocarcinoma. You refer the patient to a gynecologic oncologist for evaluation and
management.

What is the most likely intervention to be recommended? C


a. Total abdominal hysterectomy 

b. Repeat serum β-hCG surveillance
c. Chemotherapy 

d. Whole pelvis radiation 

e. Pulmonary wedge resection 


54 A 68-year-old woman presents with postmenopausal bleeding (PMB) to the gynecology


oncology clinic. An endometrial biopsy reveals a type 2 endometrioid adenocarcinoma. She
subsequently undergoes a total abdominal hysterectomy, bilateral salpingo-oophorectomy
and sentinel lymph node biopsy. You review her in the clinic with a histology report of
endometrial cancer with myometrial invasion of more than 50% and pelvic lymph node
involvement.

Choose the appropriate the International Federation of Gynecology and Obstetrics (FIGO) E
stage for endometrial cancer below:
a.Stage IB

b.Stage II
c.Stage IIIA
d.Stage IIIB
e.Stage IIIC

55 A 39-year-old woman presents for her annual examination. She inquires whether she should
receive the HPV vaccine.

For which patient population is the HPV vaccine FDA approved? C


a. Women of all ages
b. Only women who are virginal and have never had an abnormal Pap smear
c. Girls and women aged 9 to 26 years
d. Men, for the prevention of female cervical dysplasias and cancer

e. For pregnant or lactating women

56 A 59-year-old P4 presents to you with a complaint of LLQ pain, intermittent nausea,


abdominal pressure, and bloating. She have a history of obesity, right breast cancer, and
hypertension. She used to use OCP as a contraception method for 6 years and then perform
tubectomy after delivery her last child. Her family history is notable for premenopausal breast
cancer in her mother and maternal aunt. She had a pelvic ultrasound showing a normal
appearing uterus with a 8-cm left ovarian mass containing internal septations and papillary
growth. She has moderate ascites and her Ca-125 was 917.

Which of the following is associated with an increased risk of ovarian cancer? A


a. Personal history of breast cancer 

b. Multiparity 

c. History of OCP use for 6 years
d. Tubal ligation 

e. Breastfeeding

57 A patient wishes you to explain the concept of cervical intraepithelial neoplasia (CIN) III,
which has been diagnosed from her cervical biopsy taken. She had a LGSIL pap result with
HPV positive (Type 16 and 18).

What can you correctly tell her about CIN III? 
 B


a. It is an invasive cancer

b. It includes carcinoma in situ (CIS)

c. It requires no further treatment
d. It is due to a bacterial infection

e. It corresponds to the LGSIL shown on her prior Pap smear 


58 A 23 year old women come to your clinic for contaception counseling. She just deliver her
baby 1 month ago, and plan to give exclusive breastmilk to her baby. She also plan to use
contraception but she afraid that it will decrease her breast milk production.

Which is the correct statements regarding contraception in breastfeeding women? C


a. Depot medroxyprogesterone lowers the quality of breast milk
b. Estrogen-progestin birth control pills do affect the quality of breast milk 

c. Progestin-only birth control pills do not affect the quantity of breast milk 

d. Estrogen-progestin birth control pills do not affect the quantity of breast milk 

e. Intrauterine device (IUD) do affect the quality of breast milk 


59 A 28-year-old woman presents for care because a condom broke during sexual intercourse.
Coitus occurred 1 day ago, around her ovulation period. She does not wish to be pregnant.

Regarding her fear of pregnancy, which is the most appropriate next step in her C
management?
a. Advise her that unprotected midcycle coitus has a 10% risk of pregnancy
b. Prescribe intravaginal misoprostol (Cytotec)

c. Prescribe a brief course of levonorgestrel
d. Advise immediate douching

e. Advise her to await her next menses before taking any action 


60 A 23 year old G2P1 (no live child) woman at 8 weeks’ GA comes to your clinic for an initial
prenatal visit. Her previous pregnancy was complicated by fetal death due to uteroplacental
insufficiency.

Which of the following best describes the path of the maternal blood flow? D
a. Arteries to placental capillaries to veins
b. Intravillous spaces to arteries to veins
c. Veins to intravillous spaces to arteries
d. Arteries to intravillous spaces to veins
e. Veins to placental capillaries to arteries 


61 A 31-year-old G4P0A3 woman presents to the hospital with vaginal bleeding and abdominal
pain. She appears pale and states that she feels lightheaded when sitting up or standing.
She reports that she is currently 8 weeks’ pregnant. On arrival, her temperature is 37°C, BP
is 80/50, pulse rate is 115 beats per minute, and respiratory rate is 20 breaths per minute.
Abdominal examination reveals a rigid abdomen with rebound tenderness to palpation.
Pelvic examination reveals a small amount of vaginal bleeding, a 6-week-size uterus, and
fullness at the right adnexa. A urine β-hCG confirms that she is pregnant. A pelvic ultrasound
reveals a right-sided ectopic pregnancy as well as large amounts of fluid, thought to be blood
in the abdomen. She now has IV access and a bolus of IV fuids is being given. Her BP is
now 75/45 and her pulse rate is 120 beats per minute.

Her hematocrit returns as 25.2%. How is the next management for this patient? B
a. Proceed with emergency laparoscopic salpingectomy
b. Proceed with emergency laparotomy 

c. Administer IM methotrexate 

d. Transfuse the patient with two units of packed RBCs and transfer her to the ICU 

e. Start vasopressors and transfer the patient to the ICU

62 A patient presents for her routine prenatal visit at 30 weeks’ EGA. Her pregnancy up to now
has been uncomplicated. Her BMI is 25. Her laboratory testing is normal including a glucose
challenge screen. Anatomic ultrasound done at 12 weeks was normal and confirmed her
dating. Her fundal height is 37 cm today. A brief bedside ultrasound reveals an amniotic fluid
index (AFI) of 30 cm.

Which of the following situations is most likely to be the etiology of polyhydramnios in this C
case?
a. Renal atresia
b. Pulmonary hypoplasia
c. Duodenal atresia
d. Gestational diabetes
e. Anencephale

63 A-33 year old woman at 33 week of gestation present to clinic. Ultrasound demonstrates that
the fetus measure less than the 10th percentile, with non reactive NST.

Her biophysical profile is six out of eight. An abnormal biophysical profile (BPP) predicts D
which of the following?
a. A baby that will be small for gestational age (SGA)
b. Maternal preeclampsia
c. Meconium staining
d. Higher risk for antepartum death within 1 week
e. Placental abruption

64 Your patient is a 21-year-old nulligravida currently desiring pregnancy and attempting


conception. Her last menstrual period was 6 weeks ago. She presents with complaints of
vaginal spotting. Urine b-hCG positive, vital signs are normal, hematocrit is 36 volume
percent. From sonographic findings found the trilaminar endometrial lining. Other findings
include normal myometrium, a cul-de-sac without free fluid, and normal adnexa. Her serum
b-hCG level is 1200 miU/mL. She is planned to have another b-hCG measurement in 48
hours. She returns in 48 hours with minimal right lower quadrant pain. Her vital signs are
normal, and spotting has decrease. Her clinical and laboratory findings are unchanged
except for a serum b-hCG level now measuring 2300 miU/ mL.

What is the next best clinical step for this patient B


a. Administer methotrexate
b. Repeat transvaginal sonography
c. Perform dilatation and curettage
d. Schedule repeat serum b-hCG level in 48 hours
e. Emergency Laparotomy

65 A 30-year-old woman has a twin pregnancy at 12 weeks gestation. During her initial prenatal
care visit, you review risks for multifetal pregnancies.

Which of the following statements reflects the most frequent risks in twin pregnancies? C
a. Cesarean delivery is necessary in all of twin deliveries.
b. Shoulder dystocia occurs more in the aftercoming vertex twin, as compared to a
singleton.
c. Pregnancy-induced hypertension occurs at a higher rate than in singletons.
d. Perinatal death rate is less than that of singletons.
e. Congenital anomalies occur at the same rate as in singletons.

66 A 30-year-old G2P1 woman at 28 weeks’ GA comes to your office for a routine prenatal visit.
Her child was recently sent home with a rash and fever. She states that the child had a rash
on both cheeks and the pediatrician said it was a viral infection called fifth disease. Her baby
is moving well and denies any vaginal bleeding, abnormal vaginal discharge, or contractions
but she wonders if she needs any more testing to see if she has been affected.

What is the most likely causative organism of the child’s infection? C


a. Varicella
b. CMV
c. Parvovirus
d. Toxoplasmosis
e. Listeriosis

67 A 31 year old women recently underwent c-section due to uncontrolled blood glucose at 37
weeks of gestation. Prior the c-section, an amniocentesis and L/S test was performed. The
L/S ratio was 2:1 and phosphatidyl-glycerol was absent. She delivered a 3800gr baby boy.
Her baby was having infant respiratory distress syndrome (IRDS) and sent to NICU for
ventilation support.

What was the most likely reason of IRDS in this baby? B


a. Diabetic patients do not produce lecithin.
b. Fetal lung maturation may be delayed in maternal diabetes.
c. Foam test was not done.
d. The L/S test was done on fetal urine.
e. Maternal blood was present in the specimen.

68 A 30-year-old recently diagnosed with acquired immunodeficiency syndrome (AIDS). She


has not consume any medications yet. She presents with genital lesions that have been
present for 1 week. She reports having had a single sore 4 months ago that was not painful
and subsided without treatment after approximately 2 weeks.

Which of the following tests specific to these lesions do you expect to be positive? 
 E
a. PCR
b. Wright-Giemsa stain
c. Herpes simplex antibody assay

d. Positive skin punch biopsy staining for Candida
e. VDRL - RPR

69 A 23 year old women present to your clinic with complain of LLQ pain in every menstruation
period since 6 months ago. She has not married. From physical examination found
enlargement on LLQ. Ultrasound examination found a 8cm cyctic mass on left adnexa with
internal echo. She was given dienogest for treatment.

The treatment effect of progestins on endometriosis is secondary to which of the A


following?
a. Estrogen antagonism
b. Androgen antagonism
c. Aromatase inhibition
d. Glucocorticoid stimulation
e. Gonadotrophin Agonist

70 A 20-year-old patient is in your office to discuss the possibility of getting pregnant. She states
that she had regular menses every 28 days and last for 5 days. She uses four to five
tampons a day and denies dysmenorrhea.

The menstrual cycle is divided into which two phases when describing the endometrium? D
a. Follicular and secretory phases
b. Follicular and luteal phases
c. Proliferative and luteal phases
d. Proliferative and secretory phases
e. Atrophic and menstrual

71 A 22-year-old nulligravid patient presents with complaints of increasing dark coarse hair
growth over upper lip and chin, on the abdomen, and on her chest. She denies any change
in her voice, balding, or clitoral enlargement.

Which of the following is she most likely experiencing? E


a. Masculinization
b. Defeminization
c. Virilization
d. Androgenization
e. Hirsutism

72 A 36-year-old woman with a history of primary infertility presents with her partner for initial
prenatal visit. They had been attempting pregnancy for the past 4 years and have now
conceived through IVF. She had one embryo transferred 7 weeks ago. The patient’s medical
history is significant for rheumatoid arthritis and polycystic ovarian syndrome. She is
overweight. In reviewing their family history, she tells you that her grandmother was a twin.
Today she reports nausea and vomiting, which is worse in the morning. She has not had any
vaginal bleeding. You perform a transvaginal ultrasound and notice not one but two embryos.
There appears to be a thin dividing membrane between the two. Heart rate for each embryo
is around 150 bpm.

What stage of division in the embryonic disc does monochorionic–diamniotic twining occur? B
a. Before the differentiation of the trophoblast
b. After trophoblast differentiation and before amnion formation
c. After amnion formation
d. Not until after day 15 of development
e. After day 15 of development

73 A pair of monochorionic twins presents at 21 weeks' gestation with sonographic findings that
suggest twin-twin transfusion syndrome. There is significant growth discordance, no bladder
is visualized in the smaller twin, neither twin has ascites or hydrops, and umbilical Doppler
studies are normal.

What would be the assigned Quintero stage? B


a. Stage I
b. Stage II
c. Stage III
d. Stage IV
e. Stage V

74 You are providing prenatal care to a 24-year-old G1P0 woman at 17 weeks GA by LMP. She
has had a relatively normal pregnancy thus far. She has an obese BMI, otherwise without
medical or surgical history. She presented to prenatal care at 15 weeks, and so missed first-
trimester screening. She undergoes the quad screen and has an elevated level of maternal
serum alpha-fetoprotein (MSAFP).

Given the elevation in MSAFP, her pregnancy is at increased risk for which of the following? B
a. Gestational diabetes
b. Gastroschisis
c. Down syndrome
d. Klinefelter syndrome
e. Potter Syndrome

75 After a prolonged second stage, a 28-year-old woman delivers the vertex with an immediate
turtle sign with the head retracting against the perineum. McRobert’s maneuver does not
affect delivery.
Which of the following would be a helpful maneuver in managing this shoulder dystocia? D
a. Fundal pressure
b. Internal podalic version
c. Increased maternal pushing effort
d. Wood’s screw maneuver
e. Ritgen maneuver

76 A 36-year-old G2P1 with one prior cesarean delivery presents at 36 weeks’ gestation with
active vaginal bleeding and now requires emergency cesarean hysterectomy due to placenta
previa with accreta.

Compared with patients who have elective surgery, this woman is at increased risk or which B
of the following complications?
a. Bowel injury
b. Urinary tract injury
c. Venous thromboembolism
d. External iliac vessels injury
e. Hypogastric nerve injury

77 A 30-year-old G1P0 woman at 40 weeks and 4 days presents to labor and delivery unit with
second stage of labor. She pushes the head to the perineum and you deliver the baby
without complication.

You examine her for lacerations. Classification of perineal tear caused by either tearing or B
episiotomy in which injury to the perineum involves less than 50% of the external anal
sphincter thickness torn, is classified as
a. Second degree
b. Third A degree (3A)
c. Third B degree (3B)
d. Third C degree (3C)
e. Fourth degree

78 A 33-year-old G3P2 woman presents for routine prenatal care at 37 weeks. Today she
reports good fetal movement and denies leaking fluid or contractions. During your
examination you measure the fundal height at an appropriate 32 cm, and fetal heart tones
located in the upper aspect of the uterus. A bedside ultrasound reveals frank breech
presentation.

Which of the following is the next best step in management of this patient? C
a. Schedule a cesarean delivery for 38 weeks
b. Return visit in 1 week to reassess fetal position
c. Schedule an external cephalic version
d. Offer induction of labor at 40 weeks
e. Perform emergency caesarean delivery
79 A 26-year-old woman and her husband are being counseled after a first-trimester
miscarriage. She has no significant medical problems. Her physical examination is
unremarkable and in counseling her you explain in lay terms that the most likely cause of her
miscarriage is aneuploidy.

Which of the following is the most common aneuploidy causing miscarriage? A


a. Trisomy 18
b. 47 XXY
c. Triploidy
d. Unbalanced translocation
e. Tetraploidy

80 A 28-year-old G1P0 woman at 39 weeks and 6 days presents to labor and delivery unit, with
regular contractions occurring every 3 minutes. Her contractions last 30 to 50 seconds. She
not sure if she’s been leaking any fluid from her vagina.

Rupture of membrane would be best supported by which of the following? E


a. Nitrazine paper remaining orange when exposed to fluid in the vagina
b. A negative fern test
c. An ultrasound with a normal AFI
d. A negative tampon test
e. Speculum examination with evidence of pooling in the vagina

81 A 16-year-old female adolescent presents to your office with primary amenorrhea. She
notably has short stature, a webbed neck, and widely spaced nipples. She has noticed that
she has not developed breasts or pubic hair like her friends at school have. She is not
sexually active. Physical examination confirms the absence of secondary sex characteristics,
and auscultation of the chest reveals a harsh systolic murmur. Pelvic examination reveals
normal appearing female external genitalia and a normal vagina, with a palpable uterus on
bimanual examination.

To confirm your suspected diagnosis, which test would you order? C


a. Pelvic ultrasound
b. Serum FSH and LH
c. Karyotype
d. Serum estradiol
e. Growth hormone

82 A 28-year-old G3P2 woman at 10 weeks’ GA comes to clinic for an initial prenatal care visit.
She has a history of thyroid cancer and had a total thyroidectomy 4 years ago. She has no
evidence of recurrent disease. She has been maintained on 50 mcg of Levothyroxine for the
past 3 years and has no symptoms of hypothyroidism. Her other pregnancies occurred prior
to the diagnosis of her thyroid cancer so she is concerned about how this pregnancy might
be managed differently.

How would you manage her dose of Levothyroxine at this initial visit? B
a. No changes
b. Increase Levothyroxine dose by 25%
c. Triple her Levothyroxine dose
d. Decrease Levothyroxine dose to 25 mcg
e. Stop Levothyroxine

83 A chronic hypertensive patient presents with complaints of decreased fetal movement. It


appears that she is at term gestation with an estimated fetal weight of 2,250 g. Concerns are
raised regarding placental reserves for oxygenating the fetus.

This can be most directly and immediately assessed by which of the following? A
a. Biophysical profile
b. A fetal ultrasound growth curve
c. Lecithin/sphingomyelin (L/S) ratio
d. Maternal alpha-fetoprotein
e. Maternal estriol production

84 A 28-year-old woman, who is 36 weeks pregnant, is seen in antenatal clinic. This is her first
pregnancy. She is HIV positive. She has been fully compliant with her HAART (highly active
antiretroviral therapy) throughout her pregnancy. Her latest serum viral load is <50
copies/mL. Apart from HIV, her pregnancy has been uncomplicated, and she has an
appropriately grown cephalic presentation fetus. She is concerned about vertical
transmission of HIV.

Throughout pregnancy, when is transmission of human immunodeficiency virus (HIV) from D


a woman to her baby most likely to occur?
a. Before 28 weeks' gestation
b. Before 32 weeks' gestation
c. Before 36 weeks' gestation
d. In the days before delivery
e. Transmission occurs equally throughout gestation

85 A 28-year-old G2P1 woman who is 17 weeks’ pregnant presents to the emergency


department with increasing right lower extremity swelling, redness, and pain. She first
noticed these symptoms 3 days ago and has tried elevation and warm compresses, which
have not helped. She has no personal or family history of blood clots. Her vital signs are
normal. Physical examination is significant for right lower extremity edema, calf tenderness
and erythema. You suspect she has a lower extremity deep vein thrombosis (DVT) and you
plan to start her on anticoagulation therapy. The lower extremity Doppler studies show a
filling defect in the right popliteal vein.

What is the best initial treatment for this patient? C


a. Warfarin
b. IV heparin
c. Subcutaneous low-molecular-weight heparin
d. Placement of an inferior vena cava filter
e. Thrombolytic therapy
86 A 22-year-old primiparous woman develops postpartum hemorrhage unresponsive to
oxytocin and uterine massage. The uterine contraction was good. Her infant was 3800
grams. She has bled 750 cc.

What is the most possible diagnosis? A


a. Laceration of cervix or vagina
b. Placenta accreta
c. Uterine inversion
d. Ruptured uterus
e. Coagulopathy

87 A 32-year-old G1P0 woman at term is admitted in labor. The patient sustained a laceration of
the perineum during delivery. It involved the muscles of the perineal body but not the anal
sphincter.

What is concerning intervention to reduce perineal trauma during childbirth? C


a. Perineal massage should be routinely performed in the second stage of labor
b. Episiotomy should be carried out for all vaginal births
c. Episiotomy should be performed to assist instrumental deliveries if necessary
d. When fetal compromise is suspected, tested effective analgesia should be offered before
undertaking episiotomy
e. Episiotomy should be routinely offered to women with previous history third- and fourth-
degree tear

88 A 17-year-old girl presents to her physician. She has a complaint of vaginal discharge. She
has been sexually active for 1 year. She does not use condoms. On urogenital examination,
she has a moderate amount of yellow mucopurulent discharge from the endocervix. There is
no cervical motion, adnexal, or uterine tenderness. Microscopy of the vaginal discharge was
normal except greater than 10 WBCs per high power field. Vaginal pH is normal. Nucleic acid
amplification test (NAAT) confirms the diagnosis of gonorrhea.

What is the best initial treatment for this patient? A


a. Ceftriaxone 125 mg IM once
b. Ceftriaxone 125 mg IM once plus azithromycin 1 g orally once
c. Doxycycline 100 mg orally twice daily for 7 days
d. Ciprofloxacin 500 mg orally once
e. Clindamycin 300 mg orally twice times daily for 7 days

89 An 70-year-old female patient with multiple, serious medical comorbidities presents to


discuss options for treatment of her high-grade prolapse. The prolapse is externalized and
becoming ulcerated from friction against her undergarments. She cannot tolerate a pessary.
Her main priority is to correct this problem, but her primary care provider has cautioned
against a lengthy procedure. She is not interested in future intercourse.

What can you offer to this patient? E


a. Nothing can be done
b. Open abdominal sacral colpopexy
c. Robot-assisted laparoscopic sacral colpopexy
d. Hysterectomy with anterior and posterior colporrhaphy, vault suspension
e. Colpocleisis

90 A 30-year-old patient complains of amenorrhea after dilation and curettage (D&C) for
postpartum bleeding. She denies any other complaints and did not require a blood
transfusion at the time of her postpartum bleed.

Which of the following is the most likely diagnosis? B


a. Gonadal dysgenesis
b. Asherman syndrome
c. Sheehan syndrome
d. Kallmann syndrome
e. Mayer-Rokitansky-Kiister-Hauser syndrome

91 A 26-year-old G3P2A0 woman come to your clinic for preconception counseling. She has a
medical history of significant for IgA glomerulonephritis. Creatinine result is 2.0. She is
otherwise healthy.

Which of the following you tell her to expect regarding her kidney function if she were to get D
pregnant?
a. It will stay the same
b. It will improve
c. She will need kidney transplant in order to have a successful pregnancy
d. It will get worse
e. It is unlikely she will need dialysis by the end of pregnancy

92 A young couple with complaint of infertility. The patient is a 35-year-old P0A0. Her husband
is a 33-year-old who has had a semen analysis, which was reported as normal. The couple
reports having unprotected intercourse for the past 14 months. On further history, the patient
reports that her periods have been quiet irregular over the last year and that she has not had
period in the last 3 months. She also reports hot flashes, vaginal dryness, and decreased
libido. The FSH level was higher than normal limit and the estrogen level was less than 10
pg/mL. You repeat the labs in 2 weeks and the findings are similar. You have the patient and
her husband come back to clinic and gently give them the diagnosis. They have many
questions of what this means in terms of their ability to achieve a pregnancy.

You let them know that their best chances of achieving a pregnancy are with D
a. Ovulation Induction with aromatase inhibitors
b. Gonadotropins/IUI therapy
c. IVF with the patient’s own eggs
d. IVF with donor eggs
e. There is no way for this patient to carry a pregnancy given the diagnosis
93 A 16-year-old teen presents to the emergency room with left lower quadrant (LLQ) pain. She
denies sexual activity. Her pregnancy test is negative. On pelvic ultrasound, she is found to
have a unilateral complex ovarian mass. Urine analysis and cervical cultures are all negative.

What tumor marker is associated with this type of tumor? B


a. Ca 19-9
b. LDH
c. hCG
d. CEA

e. Ca 125

94 You are called to check A 42-year-old P1A0 woman presents to your emergency department
with profuse vaginal bleeding. Her son is 8 year-old. She is tachycardia with normal blood
pressure. An initial workup shows a solid intrauterine mass and a serum β-hCG level of 300
mIU/mL. You decide to proceed with endometrial sampling, which is uncomplicated. Final
surgical pathological diagnosis is consistent with Placental Site Trophoblastic Tumor (PSTT).

Which is typically the primary treatment of choice for PSTT? A


a. Hysterectomy
b. Systemic chemotherapy
c. Directed external beam radiation therapy
d. Serum β-hCG surveillance
e. Hysteroscopy, D&C

95 A 32-year-old female P0A0 presents to your office for consultation. She is afraid about her
risk of breast cancer. During her history, she reports regular, monthly menses with moderate
flow. Her mother was diagnosed with breast cancer at age 42, and her paternal aunt was
diagnosed with breast cancer at age 60. Her previous gynecologist diagnosed her with
fibrocystic breast change, and this worries her. She notices pain in both breasts often. Her
breast examination shows Tanner stage 5 breasts that are symmetric and without palpable
mass or abnormal skin changes. Upon palpation of the nipples of both breasts, a clear
discharge is noted.

Which element of her history and examination is most associated with an increased risk of D
breast cancer and ovarian cancer over the general population?
a. Nipple discharge
b. Breast pain
c. History of fibrocystic breast change
d. Mother with breast cancer at age 42
e. Paternal aunt with breast cancer at age 60

96 A 26-year old woman came to labor and delivery unit due to regular contractions. According
to your examination, the head of the fetus is already engaged.

Engagement is best defined as which of the following? C


a. When the presenting part goes through the pelvic inlet
b. When the presenting part is level with the ischial spines
c. When the greatest biparietal diameter of the fetal head passes the pelvic inlet
d. When the greatest biparietal diameter of the head is level with the ischial spines
e. When the greatest diameter of the fetal presenting part passes through the narrowest
and lowest part of the maternal pelvis

97 A 18-year-old female college freshman presents to your office with secondary amenorrhea,
but she also reports galactorrhea and headache. Your laboratory findings include an
elevated prolactin of 120 ng/mL, normal FSH, and normal TSH.

What is your next step in diagnosis? C


a. Serum estradiol
b. Pelvic ultrasound
c. MRI of the head
d. Serum testosterone
e. No further testing is necessary

98 A 40-year-old nuliparous obese woman complains of abnormal vaginal bleeding of 6 months’


duration. Pelvic examination and ultrasound demonstrates an anteverted uterus and a
normal-appearing cervix. No adnexal masses are present. A serum pregnancy test is
negative, and pap smear is normal. Prolactin and thyroid-stimulating hormone (TSH) levels
are normal.

Which of the following is the most efficient next step in the evaluation of this patient? B
a. Saline-infusion-sonography (SIS)
b. Endometrial biopsy
c. Endometrial cytology
d. MRI
e. Hysteroscopy


99 During pregnancy, the placenta and fetus actively contribute to the maternal hormone levels
and impact the maternal-fetal unit physiology. A 23 year old women G2P1A0 come to your
clinic at 24 weeks of gestation.

Which of the following hormones decreases after the first trimester of pregnancy? 
 D
a. Progesterone

b. Prolactin

c. Human placental lactogen (hPL)

d. Human chorionic gonadotropin (hCG)
e. Estriol 


100 A 28-year-old P2 woman comes to the emergency department with 6 hours of increasing left
lower quadrant pain, inability to tolerate orals, and nausea. She is sexually active and uses
Depo-Provera for contraception. She was treated for gonorrhea and reports compliance with
treatment. Her temperature is 38.5°C, pulse rate is 112/min, respirations are 22/min, and BP
is 100/70 mm Hg. On examination, her abdomen is soft with left lower quadrant tenderness.
Voluntary guarding is present without rebound. Pelvic examination shows no cervical motion
tenderness or uterine tenderness. The left adnexa is exquisitely tender and fullness is
appreciated. Her WBC count is 21,000 cells/μL. Urine HCG is negative.

Which of the following is the most appropriate next step in management? A


a. Pelvic ultrasound
b. Abdominal ultrasound
c. Measurement of serum β-hCG concentration
d. Dilation and curettage
e. Discharge home on oral antibiotics

103 You are seeing a patient at antenatal clinic at 16 weeks’ gestation to make a plan for her
antenatal care. The midwife wants to know if the woman has any major risk factors for an
SGA fetus.

Which of the following is a major risk factor for this? B


A. BMI of 19. 

B. Chronic hypertension. 

C. PAPP-A > 4 MoM on combined screening. 

D. Preeclampsia in a previous pregnancy. 

E. Smoking 10 cigarettes per day. 


104 You see a 34-year-old para 1 with a BMI of 29 in your antenatal clinic. She is at 10
weeks’gestation with a confirmed intrauterine pregnancy and is taking folic acid 400 mcg
daily. In her last pregnancy, 3 years ago, she developed preeclampsia at 37 weeks’
gestation. She was induced and spontaneously delivered a healthy female at 37+2 weighing
2990 g. She has no additional risk factors.

Which of the following would be the best initial antenatal management plan? B
A. Aspirin 75 mg from 18 weeks’ gestation until delivery, serial symphysis fundal height
at each antenatal visit plotted on a customized growth chart, uterine artery Doppler
at 20 to 24 weeks’ gestation and, if this is normal, scan for fetal size and UA
Doppler in the third trimester. 

B. Aspirin 75 mg from 12 weeks’ gestation until delivery and serial symphysis fundal
height at each antenatal visit plotted on a customized growth chart. 

C. Aspirin 75 mg from 12 weeks’ gestation until delivery, serial symphysis fundal height
at each antenatal visit plotted on a customized growth chart, serial ultrasound
measurement of fetal size and assessment of fetal well-being with UA Doppler from
26 to 28 weeks’ gestation.
D. Aspirin 75 mg from 12 weeks’ gestation until delivery, serial 
symphysis fundal height
at each antenatal visit plotted on a customized growth chart, uterine artery Doppler
at 20 to 24 weeks’ gestation and, if this is normal, scan for fetal size and UA
Doppler in third trimester. 

E. Aspirin 75 mg from 12 weeks’ gestation until 34 weeks’ gestation and serial
symphysis fundal height at each antenatal visit plotted on a customized growth
chart. 


105 A primigravida at 20 weeks’ gestation attends obstetric triage feeling short of breath. She has
a monochorionic diamniotic twin pregnancy. She has not attended for antenatal care since
her dating scan at 14 weeks’ gestation where chorionicity was confirmed, after which she
separated from her partner. Chest examination is unremarkable, but her SFH is 32 cm. An
ultrasound scan is performed, revealing that both twins are alive. Twin 1 has polyhydramnios
with a DVP of liquor measuring 9.5 cm and bladder visible. Twin 2 has oligohydramnios with
a DVP of liquor measuring 0.5 cm. Despite an hour of scanning, it is not possible to identify a
bladder in twin 2. Both twins have end diastolic flow present on UA Doppler. A diagnosis of
suspect twin–twin transfusion syndrome is made.

What Quintero stage would this be classified as? B


A. I.
B. II.
C. III.
D. IV.
E. V.

106 When aspirin is used to reduce risk of preeclampsia in woman at high risk, at what gestation B
should it be commenced for maximum efficacy?

A. Before 12 weeks 

B. Before 16 weeks
C. Before 20 weeks
D. Before 24 weeks
E. Before 28 weeks 


107 When calcium supplementation is used to reduce the risk of preeclampsia in women at high C
risk, at what gestation should it be commenced?


A. 12 weeks 

B. 16 weeks
C. 20 weeks
D. 24 weeks
E. 28 weeks 


108 A 39-year-old para 0 who is 12+2 by last menstrual period attends for dating scan. She is
keen to have screening for aneuploidy. The CRL is found to be 89 mm.

What would be the best management with regard to dating the pregnancy and screening for C
Down syndrome?
A. Date the pregnancy using biparietal diameter and arrange quadruple test for Down
syndrome screening. 

B. Date the pregnancy using CRL, measure the nuchal translucency and use the
combined test to screen for Down, Edward’s and Patau’s syndromes. 

C. Date the pregnancy using head circumference and arrange quadruple test for Down
syndrome screening. 

D. Date the pregnancy using head circumference, measure nuchal translucency and
use the combined test to screen for Down, Edward’s and Patau’s syndromes. 

E. Date the pregnancy using head circumference, offer amniocentesis to screen for
Down, Edward’s and Patau’s syndromes. 


109 A 32-year-old para 0 undergoes IVF outside the UK and has a trichorionic triamniotic triplet
pregnancy. She is concerned about how she will cope with her pregnancy towards her
due date and asks the latest she is likely to be delivered by.

When should women with trichorionic triamniotic triplets be offered elective delivery, in the C
absence of prior complications or labour?
A. From 32+0 weeks’ gestation, after a course of antenatal corticosteroids. 

B. From 35+0 weeks’ gestation. 

C. From 35+0 weeks’ gestation, after a course of antenatal 
corticosteroids
D. From 36+0 weeks’ gestation, after a course of antenatal 
corticosteroids. 

E. From 36+0 weeks’ gestation. 


110 A 35-year-old para 1 at 32 weeks’ gestation is being monitored for an SGA fetus after her
midwife plotted symphysis fundal height on a customized growth chart suggested static
growth. Her antenatal care has previously been uneventful other than a course of
steroids at 26+3 weeks’ gestation during an admission for threatened preterm labour.
Her first delivery was 3 years ago at 39 weeks with the delivery of a healthy male infant
weighing 2950 g. She has had tuberculosis as a child living in India, but is otherwise fit
and well. Ultrasound scan today reveals the fetal abdominal circumference to be around
the 7th centile on a customized chart, with an UA pulsatility index greater than 2
standard deviations from the mean for gestational age. End diastolic flow is present.

What would be the best initial follow-up plan for her? E


A. Course of antenatal steroids, daily UA Doppler and weekly abdominal circumference or
estimated fetal weight.

B. Course of antenatal steroids, twice weekly UA Doppler and weekly abdominal
circumference or estimated fetal weight.
C. Course of antenatal steroids, twice weekly CTG, weekly UA Doppler and weekly
abdominal circumference or estimated fetal weight.
D. Twice weekly CTG, weekly UA Doppler and weekly abdominal circumference or estimated
fetal weight.
E. Twice weekly UA Doppler with once weekly abdominal circumference or estimated fetal
weight.

111 A 25-year-old gravida 1 with a twin pregnancy has just had a 24-week ultrasound scan at the
antenatal clinic. The report shows twin 1 with a DVP of liquor measuring 1.4 cm and twin
2 with a DVP of 10.8 cm. Her 12-week scan report says ‘ T sign clearly seen’.

What is the most likely diagnosis? E


A. Chromosomal abnormality twin 1.
B. Cytomegalovirus.

C. Discordant fetal growth.
D. Twin reversed arterial perfusion.
E. Twin-to-twin transfusion syndrome.

112 A 32-year-old primigravida at 33+6 weeks’ gestation has been under close surveillance after
a diagnosis of an SGA fetus. She attends for ultrasound in the morning clinic. Fetal
abdominal circumference remains less than the 10th centile, DVP of liquor measures
1.2 cm and the UA Doppler shows reversed end diastolic flow. The CTG is normal. A
course of antenatal corticosteroids was completed 2 days ago. She had a large
breakfast 4 hours ago. The labour ward is busy, an elective caesarean section has just
been commenced, but you have access to open an emergency second operating
theatre. She is keen for a vaginal delivery.

What would be the best management plan you would advise for her? C
A. Commence induction on the antenatal ward. 

B. Commence induction on the labour ward under close
surveillance. 

C. Deliver by caesarean section today. 

D. Deliver by category 1 caesarean section. 

E. Repeat ultrasound and umbilical Doppler twice weekly, aiming to 
deliver at 37 weeks. 


113 A primigravida aged 26 is admitted with threatened preterm labour at 30 weeks and seeks
counselling with regards to antenatal corticosteroids.


What are the three recognised fetal benefits associated with antenatal corticosteroid D
administration in the case of premature delivery?
A. Reduced respiratory distress syndrome, reduced incidence of hypoglycemia, reduced
neonatal death rates
B. Reduced respiratory distress syndrome, reduced VII nerve damage, reduced incidence of
hypoglycemia 

C. Reduced respiratory distress syndrome, reduce incidence of pneumothorax formation,
reduced retinal disease of prematurity 

D. Reduced respiratory distress syndrome, reduced intraventricular haemorrhage reduced
neonatal death rate
E. Reduced respiratory distress syndrome, reduced intraventricular haemorrhage reduced
necrotising enterocolitis rates

114 A gravida 2 Para 0 + 1 molar pregnancy is diagnosed with Rhesus isoimmunisation.


Doppler assessment of which vessel is used to monitor fetal anaemia during pregnancy? A
A. Middle cerebral artery
B. Umbilical artery

C. Umbilical vein

D. Uterine artery
E. Uterine vein
115 A woman with a monchorionic diamniotic twin pregnancy at 25 weeks gestation is assessed
at the regional fetal medicine service. She is found to have severe TTTS (Quintero
stage III).


What is the optimal treatment? B


A. Amnioreduction

B. Fetoscopic laser ablation

C. Septostomy

D. Termination of the donor twin

E. Termination of the entire pregnancy

116 A primigravida is seen for booking. She is 40 years and has conceived through IVF.
Ultrasound scan has confirmed a twin pregnancy. Her BMI is 36kg/m2.


What treatment would you advise to reduce the risk of preeclampsia? A


A. Aspirin 75 mg daily

B. Folic acid 5 mg/day

C. Low molecular heparin 40 mg subcutaneously daily
D. Low salt diet

E. Vitamin C & E

117 A community midwife requests advice with regard to induction of labour for a woman who is
currently 40 weeks gestation. She has had 2 previous vaginal deliveries at 38 and 39
weeks. The pregnancy has been uncomplicated.

What would you advise?
 B


A. To arrange induction as soon as possible

B. To arrange induction between 41–42 weeks and discuss membrane sweep
C. To arrange induction with amniotomy at 41 weeks
D. To continue expectant management even after 42 weeks with 
fetal surveillance until
labour commences 

E. To observe for 10 days and call again if the mother had not 
delivered 


118 A gravida 3 Para 2 is diagnosed with an anterior placenta reaching to the ostium at 20
weeks. She has had 2 previous caesarean sections.

What further investigation would you arrange?
 A


A. Colour flow Doppler scan at 32 weeks
B. MRI scan at 36 weeks

C. No further investigations

D. Transvaginal scan at 32 weeks

E. Ultrasound scan at 36 weeks for placental localisation
119 A multiparous woman is admitted to a delivery suite at 37 weeks gestation. She has been
feeling unwell for the last 48 hours. She gives history of flu-like symptoms with cough,
abdominal pain and watery vaginal discharge. Her temperature is 38, pulse 110 per
minute, Respiratory rate 24 per minute. You have made a diagnosis of sepsis and
antibiotics have been commenced after blood culture. Her serum lactate is 4 mmol/l.


What would be recommended for immediate intravenous fluid resuscitation? C


A. Colloid 500 ml

B. Crystalloid 1000 ml

C. Crystalloid 30 ml/kg body weight
D. Crystalloid 50 ml/kg body weight

E. CVP line and fluid as required

120 A woman who is HIV positive attends antenatal clinic at 36 weeks gestation. She has an
uncomplicated pregnancy.


At what plasma viral load could vaginal delivery be recommended? A


A. Less than 50 HIV RNA copies/ml
B. Less than 100 HIV RNA copies/ml
C. Less than 150 HIV RNA copies/ml
D. Less than 200 HIV RNA copies/ml
E. Less than 250 HIV RNA copies/ml

121 A woman attends her first ultrasound scan in pregnancy.

What is the maximum crown rump length (CRL) that is accurate for dating before you D
measure gestational age by head circumference (HC)
A. 14 mm
B. 24 mm
C. 44 mm
D. 84 mm
E. 94 mm

122 What is the maximum crown rump length (CRL) that is accurate for dating before you D
measure gestational age by head circumference (HC)
A. 14 mm
B. 24 mm
C. 44 mm
D. 84 mm
E. 94 mm

123 A 25-year-old woman attends the Early Pregnancy Unit with vomiting and bleeding. An
ultrasound scan is performed, which is strongly suggestive of a molar pregnancy.

What is the optimal method of uterine evacuation? D


A. Medical evacuation with Methotrexate

B. Medical evacuation with Mifepristone and Misoprostol
C. Medical Evacuation with Oxytocin

D. Suction curettage
E. 
Total Hysterectomy

124 A 30-year-old woman attends the early pregnancy unit with a positive pregnancy test and
some lower abdominal pain. Her last menstrual period was approximately 8 weeks ago,
but her menstrual cycle is irregular. A transvaginal ultrasound scan is organised, which
demonstrates an intrauterine gestation sac with fetal pole and yolk sac, but no fetal
heartbeat is identified. The crown-rump length (CRL) is 6 mm.

What is the correct course of action?
 E


A. Inform the woman that she has a missed miscarriage and arrange surgical evacuation
B. Inform the woman that she has a missed miscarriage and 
arrange medical evacuation
with misoprostol 

C. Inform the woman that she has a missed miscarriage and 
arrange medical evacuation
with mifepristone and misoprostol 

D. Inform the woman that the pregnancy appears normal and no 
further intervention is
required 

E. Inform the woman that the viability of the pregnancy is uncertain and arrange for a further
scan in 7–10 days 


125 A woman underwent a surgical evacuation of the uterus following a failed intrauterine
pregnancy.
The products of conception were sent for histological analysis and a
diagnosis of complete molar pregnancy was made. The woman was referred to the
regional trophoblastic disease centre for follow-up, and subsequently required treatment
with single-agent chemotherapy.
She returns to clinic after completion of treatment as
she wishes to conceive again.

How long should she wait? D


A. 1 month

B. 3 months

C. 6 months
D. 12 months
E. 18 months

126 A 26-year-old patient presents with left iliac fossa pain and has a 6 week period of
amenorrhoea. The patient is clinically stable. An ultrasound confirms the presence of a
left-sided ectopic pregnancy. There is a 2x2x2 cm pool of free fluid in the pouch of
Douglas and the serum βHCG level is 3500 IU/l.
What is the recommended management? A
A. Left Laparoscopic salpingectomy, if the contralateral tube is healthy
B. Left Laparoscopic Salpingotomy 

C. Left Salpingectomy via a laparotomy 

D. Methotrexate 

E. Repeat the βHCG levels and manage accordingly 


127 A woman has attended the gynaecology clinic to discuss a diagnosis of a molar pregnancy.

What is the definitive method of diagnosis? B


A. Clinical assessment

B. Histologic analysis of tissue

C. Serum β-HCG levels
D. Ultrasound

E. Urine β-HCG levels

128 A 28-year-old patient attends an early pregnancy unit for a reassurance scan at 8 weeks’
gestation. Transvaginal views show a mean sac diameter of 3.5 cm and a CRL of 10
mm with no visible heartbeat. The internal os appears closed on scan and there is no
vaginal bleeding.

What are the scan findings likely to suggest? A


A. Early pregnancy demise. 

B. Incomplete miscarriage. 

C. Inevitable miscarriage. 

D. Normal ongoing intrauterine pregnancy. 

E. Pregnancy of indeterminate viability. 


129 A 28-year-old woman is admitted with severe right-sided lower abdominal pain. Her pulse is
90 bpm with a BP of 110/70 mm Hg and a transvaginal ultrasound scan shows a 2.5 cm
complex right adnexal mass. There is colour flow on Doppler but no free fluid in the
pouch of Douglas. A pregnancy test is positive with beta hCG of 1400 IU/L, and there is
no evidence of an intrauterine pregnancy. The serum progesterone is 24 nmol/L.

What is the most appropriate next step in her management? A


A. Diagnostic laparoscopy with or without surgery. 

B. Expectant inpatient management. 

C. Expectant outpatient management. 

D. Intramuscular methotrexate injection. 

E. Computed tomographic scan of the pelvis. 


130 A 20-year-old para 0 attends for an early pregnancy scan at 12 weeks. An embryo with a
CRL of 20 mm is identified with no fetal heart action seen. She opts for medical
management.

What is the appropriate next step in management? D


A. Mifepristone followed 12 hours later by misoprostol 400 mcg. 

B. Mifepristone followed 24 hours later by misoprostol 800 mcg. 

C. Mifepristone followed 36 hours later by misoprostol 800 mcg. 

D. Misoprostol 400 mcg. 

E. Misoprostol 1200 mcg. 


131 A 29-year-old patient presents with abdominal distension and vaginal bleeding at 12 weeks’
gestation. A transvaginal ultrasound scan suggests a molar pregnancy with bilateral
enlarged multicystic ovaries. The cysts are thin walled and have clear contents. There is
no obvious free fluid in the pelvis.

There is no obvious free fluid in the pelvis. What is the most likely diagnosis? E
A. Mucinous cystadenoma. 

B. Ovarian cystadenofibroma. 

C. Serous cystadenoma. 

D. Struma ovarri. 

E. Theca lutein cyst. 


132 A 32-year-old gravida 2 Para 1 has been transferred from a midwifery-led unit for lack of
progress in labour at 4 cm. Her previous baby weighed 3100 g and was a normal
delivery at 38 weeks gestation. On admission, her observations are normal and the
cardiotocography (CTG) was reassuring. The midwife who examined her has diagnosed
a complete breech presentation and this is confirmed on scan. The woman is very keen
to have a vaginal delivery and decision has been taken to allow labour to continue. After
2 hours, there is no progress in labour and the CTG has become suspicious.

What is the most appropriate action? A


A. Advise emergency caesarean section
B. Augment labour with syntocinon

C. Continue observation for one hour

D. Discuss ECV with the mother
E. Perform fetal blood sampling

133 A Gravida 4 Para 3 (three normal deliveries at term) is admitted in preterm labour at 36 + 5
days. She is known to have polyhy- dramnios but relevant antenatal investigations have
been normal. An ultrasound scan at 36 weeks gestation had revealed the estimated
fetal weight to be just below the 10th centile on a customized growth chart.
On
examination, the cervix was 4cm dilated with intact mem- branes and a high presenting
part. Five minutes after admission there is spontaneous rupture of membranes and the
CTG shows fetal bradycardia.
What needs to be excluded by a prompt vaginal examination? C
A. Amniotic fluid embolism
B. Breech presentation

C. Cord prolapse
D. Placental abruption

E. Shoulder/arm prolapse

134 A 42-year-old woman is 39 weeks gestation in her second pregnancy having had a prior
emergency caesarean section for fetal distress three years earlier. She is keen to give
birth vaginally but is requesting induction of labour because of concerns regarding the
increased risk of perinatal mortality associated with her age.

What is the most appropriate method of induction to minimise the risk of uterine rupture in E
labour?
A. Amniotomy and oxytocin
B. Dinoprostone

C. Misoprostol

D. Oxytocin alone
E. Transcervical Foley catheter

135 A 19-year-old woman is admitted at 34 weeks and 4 days gestation in her second pregnancy
with spontaneous rupture of membranes and painful uterine contractions. Her first
pregnancy resulted in a spontaneous preterm birth at 32 weeks gestation. On
examination, the cervix is fully effaced and 6 cm dilated.

What is the most appropriate treatment? C


A. Atosiban 6.75 mg intravenously

B. Benzylpenicillin 1.2 g intravenously

C. Betamethasone 12 mg intramuscularly
D. Nifedipine 20 mg orally

E. Pethidine 100 mg intramuscularly

136 A primigravida is in spontaneous preterm labour at 35 + 1 weeks of gestation. She has


progressed satisfactorily in labour and has been pushing for ten minutes. Fifteen
minutes prior to pushing, a fetal blood sampling had been performed due to a suspicious
CTG and the result was normal. You have been asked to attend as the CTG shows
prolonged bradycardia. You are not able to feel the fetal head abdominally and the
vertex is at +2 station and is less than 45∘from the occipito-anterior position.

What is the most appropriate course of action?
 B


A. Expedite delivery by performing a caesarean section
B. Expedite delivery with low mid-cavity forceps
C. Expedite delivery with rotational forceps

D. Expedite delivery with Ventouse

E. Repeat fetal blood sampling

137 A gravida 3 Para 2 is diagnosed with anterior placenta reaching to the os at 20 weeks
gestation. She has had 2 previous caesarean sections. Further imaging with colour flow
doppler at 32 weeks has confirmed major placenta praevia and placenta accreta.

What would be the recommendation for delivery? B


A. Category 3 caesarean section as soon as possible
B. Elective caesarean section at 36–37 weeks
C. Elective caesarean section at 38 weeks
D. Elective caesarean section at 39 weeks
E. Elective caesarean section at term

138 An emergency buzzer has been activated for shoulder dystocia. You are instructing two
junior midwives to assist you in delivery with McRoberts’ maneuvre.


What would you ask them to do? A


A. Hyperflex and adduct maternal thighs onto her abdomen
B. Hyperflex and adduct maternal thighs on her abdomen

C. Perform an episiotomy

D. Place the mother in a lithotomy position
E. Provide fundal pressure

139 You are asked to assess a woman’s perineum after a vaginal delivery. There is an extensive
tear disrupting the superficial muscle and 70 % of the external anal sphincter. There is
no disruption of the internal anal sphincter.

How would you classify this perineal trauma? C


A. Second degree tear
B. 3a tear

C. 3b tear

D. 3c tear
E. Fourth degree tear

140 A 40-year-old woman is diagnosed with acute myocardial infarction (AMI) at 36 weeks
gestation in her second pregnancy, she is clinically stable. She had a previous normal
vaginal delivery at term in her local hospital.

What is the most appropriate plan for timing and mode of delivery? D
A. Antenatal steroids and delivery by elective caesarean section in 
her local hospital
B. Await spontaneous labour and delivery in her local hospital 

C. Immediate transfer to a high risk obstetric unit and urgent delivery by caesarean section 

D. Induction of labour at 38–39 weeks gestation in a high risk obstetric unit with intensive
care expertise 

E. Induction of labour in her local hospital at 37 weeks gestation

A 40-year-old para 3 is delivered by SVD, and oxytocin 10 IU is given intramuscularly. During


cord traction the woman screams in severe pain, the uterus is no longer palpable
abdominally and the uterine fundus can be felt inverted in the vagina. The emergency
buzzer is pressed.

What is the next immediate step that should be performed? C


A. Administer tocolytic. 

B. Hydrostatic pressure with warm sodium chloride. 

C. Immediate manual replacement and simultaneous resuscitation. 

D. Immediate transfer to theatre for general anaesthetic and manual 
replacement in theatre.
E. Verbal consent for Huntingdon’s procedure. 


141 A 28-year-old primigravida spontaneously labours at 40+6 weeks’ gestation. The first stage
of labour is augmented at 5 cm labour and lasts for 11 hours. After 1 hour of passive
second stage, she pushes for 2 hours and is exhausted. On examination the fetus is
cephalic with 2/5 of the head palpable per abdomen. The cervix is fully dilated, direct OP
position with 2+ caput, 3+ moulding and station −1. She is contracting strongly at 4:10.
The CTG is normal, and the epidural is working well.

Which would be the best management? A


A. Caesarean section.

B. Continue pushing, reassess in 1 hour.
C. Trial of OP delivery with nonrotational forceps in theatre.
D. Trial of Keilland’s forceps in theatre.

E. Trial of rotational ventouse delivery in theatre.

142 You are asked to see a 28-year-old woman who is day 2 following a caesarean section for
unsuccessful induction of labour at 37 weeks’ gestation. She was diagnosed with mild
preeclampsia at 36 weeks’ gestation. Her blood pressure has been 155/100 mm Hg on
two occasions today, although she is asymptomatic and her deep tendon reflexes are
normal. She is breast feeding. You decide to start her on an antihypertensive agent.

Which of the following antihypertensive agents has insufficient evidence on infant safety to A
recommend or use in breast feeding mothers?
A. Amlodipine.
B. Atenolol.

C. Captopril.
D. Enalapril.
E. Metoprolol.
143 A 25-year-old woman attends the Early Pregnancy Unit with nausea and vomiting.

Which B vitamin has been shown to be effective in the reduction of nausea and vomiting of C
pregnancy (NVP)?

A. Cobalamin

B. Folic acid
C. Pyridoxine
D. Riboflavin
E. Thiamine

144 A 32-year-old primigravida commences a planned delivery in a rural stand-alone midwifery


unit. After 8 hours of established labour, the second stage of labour is diagnosed. The
woman develops an urge to push 1 hour later and she commences pushing. After 30
minutes late decelerations are heard on intermittent auscultation. On examination the
fetus is cephalic, 2/5 palpable per abdomen, fully dilated, direct OP and at station spines
−1. There is 3+ caput and 3+ moulding. A decision is made for transfer to hospital,
although this is delayed because of treacherous snow-covered and ice-covered roads.
On arrival at hospital 3 hours later the CTG is severely abnormal,with examination
findings unchanged and a category 1 caesarean section is performed. Ten minutes after
delivery resuscitation is stopped for a few seconds while the neonate is reassessed. The
fetus is still extremely floppy, with no pulse, no response to stimulation and no
spontaneous breathing. It is blue.

What is the infant’s 10-minute Apgar score? A


A. 0.
B. 1.
C. 2.
D. 3.
E. 4.

145 A 33-year-old primigravida with asthma delivers a live infant at term. There was no
meconium. The infant makes no spontaneous attempt at breathing and is floppy. It is
dried, covered and assessed. Five inflation breaths are performed. There is good chest
movement on inflation. The neonate is then reassessed: there is a heart rate of around
50 bpm although still no breathing. Senior assistance is summoned and en route.

What is the next immediate step? B


A. Chest compressions and ventilation breaths in ratio 1:1. 

B. Chest compressions and ventilation breaths in ratio 3:1. 

C. Chest compressions and ventilation breaths in ratio 5:1. 

D. Chest compressions and ventilation breaths in ratio 15:1. 

E. Chest compressions and ventilation breaths in ratio 30:2. 


146 A 39-year-old para 2 presents with an 8-month history of pelvic pain. She has a previous
vertical abdominal incision for a ruptured appendix and undergoes a diagnostic
laparoscopy using closed-entry technique at Palmer’s point.

Which of the following describes the correct anatomic position of Palmer’s point? A
A. 3 cm below the left costal margin in the midclavicular line. 

B. 3 cm below the left costal margin in the midaxillary line. 

C. 3 cm below the right costal margin in the midclavicular line. 

D. 3 cm below the right costal margin in the midaxillary line. 

E. 3 cm above the symphysis pubis in the midline. 


147 A 28-year-old amenorrhoeic woman who wishes to become pregnant attends the fertility
clinic complaining of galactorrhoea and mild visual disturbance. Her serum prolactin
level was found to be elevated. An MRI scan of the head is performed, which showed
the presence of a macroprolactinoma, but without supracellar extension.


What is the most appropriate first line management?
 A


A. Bromocriptine
B. Cabergoline

C. Quinagolide

D. Radiotherapy
E. Trans-sphenoidal surgical excision of the prolactinoma

148 A woman with tubal disease is advised to have IVF treatment to maximise her chances of
pregnancy. On reading the information leaflet, she is very concerned about the risks of
ovarian hyperstimulation syndrome.

What is the chance of developing severe ovarian hyperstimulation syndrome (OHSS) and B
requiring hospitalisation in women undergoing controlled ovarian hyperstimulation?

A. 0.1–0.2 %
B. 1–3%
C. 8–10%
D. 15–20%
E. 30–40%

149 A 32-year-old woman presents to the gynaecology clinic with galactorrhoea and secondary
amenorrhoea. A serum prolactin level is measured and found to be elevated.


What is the main mechanism by which hyperprolactinemia causes secondary amenorrhoea? D


A. Disruption of granulosa cell development

B. Induction of atrophic changes in the endometrium

C. Inhibition of follicle stimulating hormone (FSH) pulsatility
D. Inhibition of Luteinising hormone (LH) pulsatility
E. Inhibition of meiosis in the developing oocyte
150 A 55-year-old woman presents with a first episode of Post Menapausal Bleeding. A
transvaginal ultrasound scan showed an endometrial thickness of 3.8mm.


What is the most appropriate management plan? E


A. Book a repeat transvaginal scan in 6 months

B. Book for an out-patient hysteroscopy to look at endometrial cavity and obtain endometrial
sample

C. Perform a dilatation and curettage
D. Perform an endometrial sampling to rule out endometrial cancer
E. Reassure, follow expectant management

151 A 60 year old undergoes hysterectomy and bilateral salpingooophorectomy for grade 1
endometrial cancer.
The final histology report confirms tumor invading the uterine
serosa.

As per the new FIGO staging of endometrial cancer, what is the stage?
 C

A. Stage IC

B. Stage IIC
C. Stage IIIA
D. Stage IIIB
E. Stage IIIC

152 Ovarian cysts are common in postmenopausal women, although their prevalence is lower
than in premenopausal women.
A 59-year-old woman is referred to the clinic with
fullness in the lower abdomen and a serum CA125 level of 64 iu/l.

What is the first line of investigation?
 D


A. Computed Tomography (CT ) of abdomen and pelvis
B. MRI of pelvis

C. PET scan

D. Transvaginal ultrasound scan of pelvis

E. USS of pelvis with Doppler

153 A 49-year-old para 3 underwent laparoscopic left salpingooophorectomy for a complex left
ovarian cyst. Histology shows a serous micro papillary borderline ovarian tumor with the
presence of DNA aneuploidy.

What is the most appropriate management plan? E


A. CT scan in 6 months time

B. Reassure and discharge

C. Right salpingo-oophorectomy
D. Total abdominal hysterectomy and right salpingo-oophorectomy
E. Total abdominal hysterectomy and right salpingo- oophorectomy, peritoneal washings,
infracolic omentectomy and exploration of entire abdominal cavity

154 A 70-year-old woman has undergone laparotomy for suspected ovarian cancer. At
laparotomy, the cancer is found to involve the left ovary and uterus and she has positive
peritoneal washings.

As per the FIGO classification for staging of ovarian cancer, what is her staging? D
A. Stage IC
B. Stage IIA
C. Stage IIB
D. Stage IIC
E. Stage IIIC

155 A 65-year-old woman presents with a history of vulval discom- fort and soreness for 6
months. On examination, there is 2.5 cm raised ulcerated area on the left labia majora
which looks highly suspicious of vulval cancer.

What is the first line of investigation? A


A. Multiple punch biopsies with adjacent normal skin
B. Sentinel lymph node biopsy

C. Urgent CT scan

D. Urgent MRI
E. Wide local excision biopsy

156 Vulval cancers account for 6% of gynaecological cancers in the United Kingdom. In 2009, a
new FIGO staging was introduced with greater emphasis on the inguino-femoral lymph
node status to understand prognosis.

What is the FIGO stage for a woman who has a 3cm vulval cancer involving the anus with A
metastases in 2 lymph nodes <5 mm?

A. Stage IIIA
B. Stage IIIB
C. Stage IIIC
D. Stage IVA
E. Stage IVB

157 A 45-year-old woman complains of intermenstrual bleeding for the past 6 months. Past
history includes 6 normal vaginal deliv- eries and hypertension and last smear was over
5 years ago. On speculum examination, there is a raised 2 cm friable area on the cervix.

What is the most likely diagnosis? A


A. Cervical cancer
B. Cervical ectropion
C. Cervical polyp

D. Cervical warts
E. Chlamydia infection

158 A 40-year-old woman with severe dyskariosis on smear underwent colposcopy and large
loop excision of transformation zone (LLETZ). Histology confirmed a moderately
differentiated squamous cell carcinoma 4mm deep and 6 mm wide. Clinical and
radiological examination confirmed organ confined disease.

What stage of cervical cancer is this? B


A. Stage IA1
B. Stage IA2
C. Stage IB1
D. Stage IB2
E. Stage IIA1

159 A 35-year-old woman is diagnosed with stage IB1 cervical squamous cell carcinoma of the
cervix on histological and clinical assessment.


What is the most appropriate radiological investigation for this patient? C


A. CT scan of abdomen and pelvis

B. CT scan of chest, abdomen and pelvis
C. MRI of abdomen and pelvis

D. PET CT scan
E. Ultrasound of abdomen and pelvis

160 A 53-year-old woman is diagnosed with stage IA1 cervical squamous cell carcinoma after
histological, clinical and radiological assessment.


What is the most appropriate management plan? D


A. Conisation

B. Modified radical hysterectomy with removal of lymph nodes
C. Radical trachelectomy

D.Total abdominal hysterectomy with bilateral salpingo-oophorectomy

E. Total abdominal hysterectomy with conservation of ovaries

161 A 25-year-old para 0, who wished to preserve future fertility, underwent a cervical conization
after an unsatisfactory colposcopy. The conization specimen demonstrated an invasive
carcinoma o the cervix infiltrating 2.5 mm below the basement membrane. There was no
evidence of lymphovascular space involvement, and the margins of the cone were free
of dysplasia or carcinoma.

What would be the best strategy of management for this patient? C


a. External beam radiotherapy. 

b. External beam radiotherapy ollowed by brachytherapy. 

c. No further therapy. 

d. Radical hysterectomy. 

e. Simple hysterectomy. 


162 A 15-year-old patient presents with left lower abdominal pain. Transvaginal and
transabdominal ultrasound scans show a 10-cm solid ovarian mass. The lactate
dehydrogenase is elevated with a normal alpha fetoprotein and human chorionic
gonadotropin.

What is the most likely diagnosis? A


A. Dysgerminoma. 

B. Embryonal cell carcinoma. 

C. Mucinous cystadenocarcinoma. 

D. Serous cystadenocarcinoma. 

E. Yolk sac tumour.

163 A 29-year-old patient is found to have high-grade squamous dyskaryosis at routine cervical
smear at 7 weeks’ gestation. Colposcopic biopsies show early invasive disease. A
subsequent cold-knife conization at 17 weeks shows microinvasive carcinoma of the
cervix with clear surgical margins.

What would be the most appropriate plan of action? E


A. Caesarean section at 34 weeks followed by radical hysterectomy. 

B. Caesarean section at 34 weeks followed by simple hysterectomy. 

C. Radical hysterectomy be ore 20 weeks ofgestation. 

D. Induction o labour at 34 weeks aiming or vaginal delivery. 

E. Vaginal delivery at term, providing there are no obstetric contraindications. 


164 An 84-year-old patient who had a previous history of vaginal hysterectomy presents with a
stage 3 vault prolapse. The patient has limited mobility and has previously had difficulty
with the use of vaginal pessaries.

What is the most appropriate treatment option? B


A. Abdominal Sacrocolpopexy

B. Colpocleisis
C. Physiotherapy
D. Sacrospinous fixation

E. Transvaginal repair with mesh

165 A patient presents with symptoms of a prolapse. On examination, the pelvic organ
quantification score is Aa 0, Ba 0, C −5, D −7, Ap −2 Bp −2 tvl 9, gh 4, pb 3.
The patient
wants her prolapse to be treated surgically.
What is the correct diagnosis and surgical treatment? B
A. Stage 1 cystocele and no treatment needed

B. Stage 2 cystocele and anterior repair

C. Stage 2 rectocele and posterior repair
D. Stage 2 uterine prolapse/cystocele and vaginal hysterectomy and anterior repair
E. Stage 2 uterine prolapse/rectocele and vaginal hysterectomy and posterior repair

166 A 39-year-old patient presents with symptoms of leakage of urine upon coughing, sneezing
and during exercise. The symptoms started following the birth of her second child 18
months ago.

What would be the first line of management? E


A. Biofeedback/Electrical stimulation

B. Bladder retraining
C. Insertion of a midurethral retropubic tape
D. Pharmacotherapy with Duloxetine

E. Supervised Pelvic floor muscle training

167 A 38-year-old patient is suffering with stress incontinence. Her BMI is 32 kg/m2 and the
patient is interested in lifestyle management for her incontinence.

What is the most important lifestyle change that you would recommend? E
A. Avoidance of caffeinated drinks
B. Exercise
C. Reduction of alcohol intake

D. Reduction of fluid intake
E. Weight loss

168 A 55-year-old patient presents with a history of urinary symptoms of urgency, increased
frequency and nocturia. The patient states that she does not have symptoms of
hesitancy and feels as though she empties her bladder completely.

What would be the first line of management? C


A. Cystoscopy

B. Neuromodulation

C. Reduce caffeine intake and start anticholinergic medication

D. Ultrasound scan to rule out pelvic pathology
E. Urodynamics

169 A 38-year-old Asian mother has delivered her fourth baby normally. She is a known Type 2
diabetic and was taking Metformin prior to pregnancy for glycemic control. From 32
weeks gestation, Isophane insulin was added twice daily in addition to Metformin to
achieve glycemic control. The woman is planning to breast feed.

What advise should be given with regard to a hypoglycemic agent in the postnatal period? C
A. Continue all the medications for the first 24 hours after delivery 
and then resume
Metformin as per prepregnancy with monitoring of blood sugar 

B. Stop all medications and follow diet control with monitoring of 
blood sugar
C. Stop Insulin and advise Metformin as per prepregnancy with monitoring of blood sugar 

D. Stop Metformin and continue Isophane insulin twice daily until breast feeding has stopped
E. Stop Metformin and continue Isophane insulin at half the dose used during pregnancy until
breastfeeding is stopped 


170 A 24-year-old presents to the A&E department with pelvic pain and vaginal discharge. On
examination she is found to be pyrexial (38°C) with moderate lower abdominal
tenderness. Vaginal examination
confirms bilateral adnexal tenderness and cervical
motion tenderness.

Which of the following clinical features are not suggestive of PID? E


A. Lower abdominal pain which is typically bilateral. 

B. Deep dyspareunia. 

C. Abnormal vaginal bleeding, including post coital, intermenstrual 
and menorrhagia. 

D. Abnormal vaginal or cervical discharge which is o en purulent. 

E. Lower abdominal pain which is typically unilateral. 


171 A 28-year-old nulliparous patient attended for routine cervical screening which has shown
borderline changes and HPV inadequate results. She is not currently sexually active and
has had normal smear results before this test.

What is the next appropriate step in management? D


A. Refer to colposcopy. 

B. Repeat cytology only in 3 months. 

C. Repeat cytology and HPV in 3 months. 

D. Repeat cytology and HPV in 6 months. 

E. Repeat cytology and HPV in 12 months. 


172 An 18-year-old nulliparous patient attends the gynaecology clinic for heavy painful menstrual
bleeding. She is in a sexual relationship with a new partner and both have tested
negative for STIs, though she has had a previously treated chlamydia infection. She
does not want any children in the near future. She takes lamotrigine for epilepsy control
and has not suffered any seizures for 3years. General and pelvic examinations are
normal, and her BMI is 30.

What is the best appropriate method of contraception in her situation? D


A. Combined oral contraceptive pill. 

B. Depot medroxyprogesterone acetate. 

C. Evra patch. 

D. Levonorgestrel-releasing intrauterine system. 

E. NuvaRing 


173 A 35-year-old nulliparous patient is HIV positive and takes efavirenz and nevirapine. She is
using a 30-mcg oestrogen combined oral contraceptive preparation as well as barrier
contraception. She finished a pill packet 10 days ago, forgot to restart again and had
unprotected sexual intercourse 2 days ago. She also missed two pills in her first week of
the previous pill packet. A chlamydia urine polymerase chain reaction test is positive.
She attends asking for emergency contraception.

What is the most appropriate method of emergency contraception in this situation? B


A. 1.5 mg levonorgestrel as soon as possible. 

B. 3 mg levonorgestrel as soon as possible. 

C. LNG-IUS as soon as possible. 

D. The copper IUD as soon as possible. 

E. Ulipristal acetate single 30 mg tablet as soon as possible.

174 A hysteroscopic tubal sterilization using fexible microinserts (Essure) is performed. The
procedure time is about 30 minutes (from insertion to removal of hysteroscope), and
there was a concern regarding possible perforation due to a sudden loss of resistance at
insertion.

What is the suggested method of confirming that the procedure has been successful? A
A. Hysterosalpingogram. 

B. Magnetic resonance imaging. 

C. Transvaginal ultrasound scan. 

D. Pelvic x-ray. 

E. Diagnostic laparoscopy. 


175 A 36-year-old para 2 who is HIV positive with a CD4 count of 73 has been recently started on
antiretrovirals. She has attended for her routine cervical smear and mentions that she
has developed an ulcer on her labia. Clinically this is vulval herpes simplex virus.

What is the best treatment regime? D


A. Oral aciclovir 200 mg five times a day for 5 days. 

B. Oral aciclovir 200 mg 7 to 10 times a day for 5 days. 

C. Oral aciclovir 400 mg five times a day for 5 days. 

D. Oral aciclovir 400 mg five times a day for 7 to 10 days. 

E. Oral aciclovir 800 mg five times a day for 5 days.

176 A 34-year-old woman with a BMI of 29, who has been taking enalapril for essential
hypertension, requests the combined oral contraceptive pill. Her BP is well controlled on
this treatment and today is 134/88 mm Hg.

To which Medical Eligibility Criteria would well-controlled hypertension be considered? C


A. 1.
B. 2.
C. 3.
D. 4.
E. 5.

177 A 35-year-old nulliparous patient is HIV positive and takes efavirenz and nevirapine. She is
using a 30-mcg oestrogen combined oral contraceptive preparation as well as barrier
contraception. She finished a pill packet 10 days ago, forgot to restart again and had
unprotected sexual intercourse 2 days ago. She also missed two pills in her first week of
the previous pill packet. A chlamydia urine polymerase chain reaction test is positive.
She attends asking for emergency contraception.

What is the most appropriate method of emergency contraception in this situation? B


Options
A. 1.5 mg levonorgestrel as soon as possible. 

B. 3 mg levonorgestrel as soon as possible. 

C. LNG-IUS as soon as possible. 

D. Copper IUD as soon as possible. 

E. Ulipristal acetate single 30 mg tablet as soon as possible 


178 A 35-year-old para 3 underwent a TAH+BSO for severe pelvic endometriosis that had not
responded to medical and conservative surgical treatment. She attends for her follow-up
appointment complaining of hot flushes and sweating.

Which HRT regime should you consider? A


A. Combined continuous oestrogen/progestogen. 

B. Oestradiol patches. 

C. Oral oestradiol valerate. 

D. Progesterone-only HRT. 

E. Topical oestriol.

179 A 25-year-old patient presents with sudden-onset LIF pain associated with nausea and
vomiting. She is sexually active, uses the Mirena intrauterine contraceptive device for
contraception and is otherwise healthy. On examination she is tachycardic, and
abdominal examination demonstrates tenderness over LIF with no rebound. Pelvic
examination shows a normal cervix with no abnormal discharge, but fullness and
tenderness on the left. The urine pregnancy test is negative.

What is the most likely diagnosis? C


A. Diverticulitis. 

B. Ectopic pregnancy. 

C. Ovarian torsion. 

D. Ruptured corpus luteum. 

E. Tubo-ovarian abscess. 


180 A 25-year-old patient suffers with headaches, mood swings, irritability, depression and
feeling out of control just before, and during, her menses. She also reports physical
symptoms including breast tenderness, bloating and headaches. Her physical and
psychological symptoms resolve completely after menstruation ceases.

Which of the following would be the best first-line management? B


A. A second-generation combined oral contraceptive pill used cyclically. 

B. Cognitive Behavioral Therapy. 

C. Continuous high-dose SSRI. 

D. Danazol 

E. Vitamin B6 20 mg daily. 

181 A 16-year-old girl presents with primary amenorrhoea. She has reached Tanner V breast and
pubic hair development and, on pelvic examination, there is a blind ending vagina. The
karyotype shows 46XX.

What is the most likely diagnosis? C


A. Congenital adrenal hyperplasia. 

B. Constitutional delay. 

C. Mayer–Rokitansky–Küster-Hauser syndrome. 

D. McCune–Albright syndrome. 

E. Complete androgen insensitivity syndrome.

182 A 25-year-old para 2 presents with a 4-year history of progressive facial hirsutism and
increasingly irregular periods. She had menarche at the age of 8. On examination she
has moderately severe hirsutism on the face and chin and was not Cushingoid in
appearance. Her BP was 136/83 mm Hg and her BMI was 23.3 kg/m2. She reports that
she was a tall child and volunteered that she has always had a small vaginal introitus
with clitoromegaly and, since puberty, a high libido.

What are you likely to find in her laboratory blood tests? D


A. Decreased serum testosterone. 

B. Elevated sex hormone-binding globulin. 

C. Elevated gonadotropins. 

D. Elevated 17-hydroxyprogesterone. 

E. Elevated TSH. 


183 A 35-year-old para 2 presents with dysmenorrhoea over the preceding 12 months. She also
reports some dyschesia (diffcultly defecating) during her periods. A transvaginal
ultrasound scan confirmed a 7-cm endometrioma. She undergoes a laparoscopic
ovarian cystectomy and excision of pelvic endometriosis.

Which is of the following should be your postsurgical management plan? C


A. COCPs for 3 months. 

B. COCPs for 3 to 6 months. 

C. LNG-IUS for 18 months. 

D. GnRH for 6 months. 

E. GnRH for 12 months with add back tibolone. 


184 A 35-year-old patient presents 6 months after a normal vaginal delivery. The pregnancy was
uncomplicated but, following delivery, a piece of placenta was apparently retained in the
uterus. She was treated with antibiotics and later underwent a dilatation and curettage
procedure. Now she presents with amenorrhoea. She is no longer breast feeding and is
concerned.

What is the most appropriate next step in her management? C


A. Hysterosalpingo-contrast sonography. 

B. Hysterosalpingogram. 

C. Hysteroscopy. 

D. Hysterosonography. 

E. Saline sonography. 


185 A 40-year-old woman with BMI 32 kg/m2 is referred to the gynaecology clinic with secondary
amenorrhoea. She has two children and her partner had a vasectomy 5 years ago.
An
ultrasound scan is performed, which shows a normal uterus with endometrial thickness
6 mm. Both ovaries have a typical polycystic appearance.

What would be the recommended management? B


A. Endometrial biopsy
B. Induction of 3-monthly withdrawal bleeds with progestagens 

C. Metformin twice daily 

D. Ovulation induction with clomiphene citrate 

E. Reassure and discharge 


186 A 20-year-old girl attends the gynaecology clinic with her mother. She presents with primary
amenorrhoea.
On examination she is tall with a BMI of 19 kg/m2 . She has normal
breast development, but a short blind-ending vagina. There is no axillary or pubic hair.

What is the most likely diagnosis? A


A. Complete androgen insensitivity syndrome
B. Klinefelter syndrome

C. Mullerian agenesis

D. Swyer syndrome
E. Turners syndrome

187 A 42-year-old woman with oligomenorrhoea and hirsutism presents to the gynaecology clinic.
She recently had a prolonged episode of vaginal bleeding, but an ultrasound scan and
endometrial biopsy performed in primary care both reported normal results. She is
obese with a BMI of 40kg/m2. She has mild hypertension but does not require
antihypertensive therapy. She has no other medical problems. Her father suffered from
Type 2 Diabetes mellitus.

What further investigation is required?
 C


A. LH:FSH ratio

B. MRI scan of pelvis
C. Oral glucose tolerance test
D. Pregnancy test

E. Serum cholesterol

188 A 21 year old presents to the gynaecology outpatient clinic with pelvic pain. The general
practitioner referral suggests possible endometriosis.


What is the estimated prevalence of endometriosis in women of reproductive age? B


A. <1%

B. 2–10%
C. 10–20%
D. 25%

E. 50%

189 A 47-year-old Para 3 who has had three previous vaginal deliveries presents with a history of
HMB that has not responded to medical treatment or the levonorgestrel-containing
intrauterine system (LNG-IUS). The patient was offered endometrial ablation but
declined.
On examination, the uterus is bulky, no masses palpable in the adnexa and the cervix
descends to about 2 cm above the hymenal ring. An ultrasound confirms the physical
examination findings.

What is the most appropriate treatment option? E


A. A combination of endometrial resection and levonorgestrel releasing-IUS
B. Laparoscopic vaginal assisted hysterectomy as it enables the surgeon to assess the
pelvic organs 

C. Subtotal hysterectomy as it avoids possible bladder injury and has a lower incidence of
sexual dysfunction 

D. Total abdominal hysterectomy as there is a lower risk of bladder injury than a vaginal
hysterectomy 

E. Vaginal hysterectomy

190 A 39-year-old woman presents to the gynaecology clinic with HMB and dysmenorrhoea. She
is otherwise fit and well.
Pelvic examination is unremarkable.
She is not keen on
hormonal methods of treatment.

What treatment would you initially recommend? C


A. Danazol

B. Etamsylate

C. Mefenemic acid
D. Norethisterone
E. Tranexamic acid

191 A 38-year-old woman is seen in the gynaecology clinic. She presented with Heavy Menstrual
Bleeding. History and examination are unremarkable and she is commenced on
tranexamic acid, to be taken during menstruation only.

Should this treatment ultimately prove to be ineffective, for how many cycles should she have A
tried it to come to this conclusion?
A. 3 cycles
B. 6 cycles
C. 9 cycles
D. 12 cycles
E. 18 cycles

192 A 21-year-old G1 now P1 just delivered after a prolonged induction of labor due to being
postdates. After the placental delivery she continues to bleed excessively. Your initial
intervention to address this bleeding is to activate the normal physiologic mechanisms.

Which of the following is the most important hemostatic mechanism in combating postpartum A
hemorrhage? 

A.contraction of interlacing uterine muscle bundles
B.fibrinolysis inhibition

C.increased blood-clotting factors in pregnancy 

D.intramyometrial vascular coagulation due to vasoconstriction
E.markedly decreased blood pressure in the uterine venules

193 A 22-year-old G3P2002 who had a hematocrit of 36% at her initial obstetrical examination at
12 weeks is found to have a hematocrit of 30% at 28 weeks when checked along with
her 1 hour glucola. Based on the indices of the red blood cells on the CBC, you
diagnose iron deficiency. She asks why that occurred since she has been taking her
prenatal vitamins.

As part of the explanation, you note that which of the following maternal measurements or A
findings is first decreased by the iron requirements of pregnancy?
A.bone marrow iron

B.hemoglobin

C.jejunal absorption of iron
D.red cell size

E.serum iron-binding capacity

194 A 25-year-old G3P2 in her sixth week of pregnancy, by last menstrual period (LMP)
calculation, has an endovaginal ultrasound examination because of vaginal bleeding.
The ultrasound confirms an intrauterine pregnancy with fetal cardiac activity present and
fetal pole length consistent with 6 weeks’ gestation. Scan of the adnexae reveals a 5-cm
simple cyst on the left ovary.

Which of the following statements is true? C


A.This patient likely has both an intrauterine pregnancy and an ectopic pregnancy.

B. This patient should be told that she will probably miscarry.

C.The ovarian cyst should not be removed.

D.First-trimester vaginal bleeding is uncommon and implies a poor pregnancy outcome.
E.This patient has a blighted ovum.

195 A patient at her 34-week prenatal visit inquires as to the estimated fetal weight. When told it
is likely 4 pounds she gets worried that with only a few more weeks that her fetus is too
small and there is a problem. While reassuring her, she is told that most of the growth of
an infant is in the last month or two of the pregnancy.

During the last month of normal pregnancy, the fetus grows at a rate of approximately which B
of the following?
A.100 g/wk
B.250 g/wk
C.500 g/wk
D.759 g/ wk
E.1,000 g/wk

196 A patient is found to be blood type A negative during her first pregnancy. She receives
antenatal RhoGAM at 28 weeks. At 32 weeks she develops severe preeclampsia and is
induced, resulting in an uncomplicated vaginal delivery. The infant does well and is
found to be A positive. The mother is found to have anti-D immunoglobulin at a titer of
1:1.
Which of the following best describes how much RhoGAM the patient should receive? C
A.no RhoGAM since she is sensitized
B.mini RhoGAM dose

C.one dose of RhoGAM

D.two doses of RhoGAM
E.three doses of RhoGAM

197 A couple bring their newborn to the pediatrician with concerns regarding possible bruises on
the infant’s leg. There is no evidence of trauma or violence. The couple had an
uncomplicated home birth. Concerns are raised regarding the infant’s coagulation
status.

Which of the following best describes typical fetal coagulation at birth? E


A.demonstrates a hypercoagulable state
B.depends solely on platelet activity until well into the first week of life when clotting factors
are activated
C.differs significantly between male and female fetuses

D.is generally the same as in an adult

E.shows significantly less clotting capabilities than in the adult

198 A woman presents for her new obstetrical visit at 12 weeks’ EGA. Her medical history is
complicated by Graves thyroiditis that has been treated with radioactive iodine a few
years prior. The patient is currently being maintained on thyroid replacement. She is
worried that this will compromise the fetus. She is told that the interaction between
maternal and fetal physiology relative to thyroid function is complex.

Which of the following is an accurate description of this interaction? D


A.Maternal thyroid hormones (T4 and T3) readily cross the placenta.
B.Maternal thyrotropin easily crosses the placenta.

C.The athyroid fetus is growth retarded at birth.

D.The fetal thyroid concentrates iodide.
E.The placenta serves as a barrier to maternal iodine crossing to the fetus.

199 The physiologic changes of pregnancy can alter many of the common laboratory tests.
During the evaluation of a patient with tachycardia, hypertension, and headache you are
considering both hyperthyroidism versus an atypical preeclampsia and draw the
following laboratory tests. To correctly interpret the results, it is necessary to distinguish
between normal versus abnormal changes during pregnancy.

Which of the following would normally be expected to increase during pregnancy? E


A.alanine aminotransferase (ALT)
B.aspartate aminotransferase (AST)
C.hematocrit

D.plasma creatinine
E.thyroxine-binding globulin (TBG)

200 A patient has an emergent cesarean section for an abruption. Because of a large anterior
placenta, the placenta was entered during the surgery. The mother is Rh-negative. The
infant appears anemic and is Rh-positive. To determine the amount of RhoGAM that
needs to be given to prevent sensitization, an estimate of the amount of fetal red blood
cells (RBCs) in the maternal circulation is necessary.

Fetal RBCs can be distinguished from maternal RBCs by which of the following? D
A.lack of Rh factor

B.lower amounts of hemoglobin
C.nucleated RBCs

D.resistance to acid elution

E.shape

201 A mother and her 16-year-old daughter present to your office because the daughter has not
yet menstruated. They are very concerned that something is wrong. By applying
principles of puberty to this patient, it is possible to determine if the teen is simply
undergoing a slightly delayed puberty versus potentially manifesting a significant
endocrine or anatomical problem.

Which of the following best describes the normal sequence of pubertal changes in the D
female?
A.maximal growth velocity, menarche, thelarche

B.maximal growth velocity, thelarche, menarche

C.menarche, maximal growth velocity, thelarche
D.thelarche, maximal growth velocity, menarche
E.thelarche, menarche, maximal growth velocity

202 A patient with an infected incomplete abortion presents to the emergency department. During
the dilatation and curettage (D&C) excessive bleeding develops that requires vigorous
curetting to control. She returns to her physician 6 months later complaining that she
has not had a menstrual cycle since. She has all the symptoms of getting ready to start
a period but never sees any bleeding.

This history implies what layer of endometrium is damaged? B


A.arteriole zone
B.basalis zone
C.compact zone
D.functional zone
E.spongy zone

Anda mungkin juga menyukai